Re: [obm-l] f(x + y) = f(x) + f(y)

2021-05-10 Por tôpico Bruno Visnadi
A princípio, não há nada que garanta que f seja derivável ou mesmo que o
limite exista para esta prova valer. Mas, de fato, se o domínio está
restrito a Q, você pode mostrar que f(x) = ax para algum a. Um caminho é
definir f(1) = a e mostrar que f(1/n) = a/n, para então chegar em f(m/n) =
ma/n.

Se o domínio fosse R, no entanto, não seguiria que f(x) = ax. O máximo que
poderia-se dizer é que se T ∈ R é tal que existe um r de modo que para todo
x ∈ T, x/r ∈ Q, existe um b tal que f(x) = bx para todo x ∈ T. Ou seja,
dentro de determinadas classes de R, vale f(x) = ax, mas o valor de a pode
mudar para cada classe.

Em qua., 5 de mai. de 2021 às 11:14, Claudio Buffara <
claudio.buff...@gmail.com> escreveu:

> f(x) = ax + b só satisfaz isso se b = 0.
> Tente com x+1, por exemplo.
> E mais: sem alguma outra condição (do tipo continuidade ou monotonicidade)
> ainda assim a expressão não implica que f(x) = ax.
>
> Abs,
> Cláudio.
>
>
> Enviado do meu iPhone
>
> > Em 5 de mai. de 2021, à(s) 09:13, joao pedro b menezes <
> joaopedrobmene...@gmail.com> escreveu:
> >
> > Eu estava fazendo um exercício de equações funcionais e me deparei
> com essa expressão. Não sei o que aconteceu, mas tive uma crise
> existencial e decidi provar que isso implica f(x) = ax + b( ou pelo menos
> acho que implica). Essa prova estaria certa?:
> > (obs: a função é definida nos racionais)
> > f(x + 0) = f(x) + f(0) => f(0) = 0.
> > f(x + h) = f(x) + f(h) ->
> > (f(x + h) - f(x))/h = f(h)/h = (f(h) - f(0))/h
> > agora basta fazer lim h -> 0 e obtemosÂ
> > f’(x) = f’(0) . Mas f’(0) é uma constante, logo f(x) = ax + b
> > (obs: tenho quase certeza que ela seria válida para os reais, porém
> como a função é limitada aos racionais, estou em dúvida)
> >
> > --
> > Esta mensagem foi verificada pelo sistema de antivírus e
> > acredita-se estar livre de perigo.
>
> --
> Esta mensagem foi verificada pelo sistema de antivírus e
>  acredita-se estar livre de perigo.
>
>
> =
> Instru�ões para entrar na lista, sair da lista e usar a lista em
> http://www.mat.puc-rio.br/~obmlistas/obm-l.html
> =
>

-- 
Esta mensagem foi verificada pelo sistema de antiv�rus e
 acredita-se estar livre de perigo.



Re: [obm-l] Probabilidade

2019-08-05 Por tôpico Bruno Visnadi
Existem 4 grupos possíveis para abrigar os 4 times nordestinos. A
probabilidade é, portanto, 4/C(16,4) ou 1/C(15, 3).

Imagine que você fixe a posição de um dos 4 times nordestinos no grupo X.
Sobram 15 times, e as chances dos outros 3 nordestinos ocuparem as 3 vagas
restantes no grupo X é 1/C(15, 3)

Em seg, 5 de ago de 2019 às 16:26, Claudio Buffara <
claudio.buff...@gmail.com> escreveu:

> Existem C(16,4) maneiras diferentes de escolher 4 times de um conjunto com
> 16 times. Em apenas uma delas os 4 times escolhidos são os nordestinos.
> Logo, a probabilidade desejada é 1/C(16,4).
>
> Outra maneira de fazer isso é:
> No de casos possíveis = 16!/(4!)^4 * 4!  (a multiplicação por 4! distingue
> o evento com os times 1,2,3,4 no grupo A do evento com os times 1,2,3,4 no
> grupo B).
> No. de casos favoráveis =  12!/(4!)^3 * 4!
>
> Probabilidade = ( 12!/(4!)^3 * 4! )/( 16!/(4!)^4 * 4! ) = 12!*4!/16! =
> 1/C(16,4).
>
> []s,
> Claudio.
>
>
>
> On Mon, Aug 5, 2019 at 3:53 PM Rodrigo Ângelo 
> wrote:
>
>> Eu cheguei a uma resposta diferente:
>>
>> (4!12!4)/16! =~ 0,002
>>
>> Acho que isso pode mudar dependendo de como é esse sorteio (estou
>> assumindo que serão sorteados os 16 times, sem reposição, e os quatro
>> primeiros ficam no primeiro grupo, os 4 seguintes no segundo grupo e assim
>> sucessivamente).
>>
>> Sobre as outras perguntas, acho que a definição do espaço amostral vai
>> depender do evento de interesse e da forma como você define esse evento,
>> mas não entendi direito a sua dúvida.
>>
>> Atenciosamente,
>> Rodrigo de Castro Ângelo
>>
>>
>> Em seg, 5 de ago de 2019 às 13:53, marcone augusto araújo borges <
>> marconeborge...@hotmail.com> escreveu:
>>
>>> Um campeonato vai ser disputado por 16 times, sendo 4 nordestinos. A
>>> primeira fase contará com 4 grupos de 4 times, determinados por sorteio.
>>> Qual a probabilidade de todos os nordestinos ficarem no mesmo grupo?
>>>
>>> Seria 4/(16!/4!4!4!4!)?
>>> Podemos considerar espaços amostrais diferentes em soluções diferentes?
>>> Se sim, quais os espaços amostrais possíveis?
>>> --
>>> Esta mensagem foi verificada pelo sistema de antivírus e
>>> acredita-se estar livre de perigo.
>>>
>>
>> --
>> Esta mensagem foi verificada pelo sistema de antivírus e
>> acredita-se estar livre de perigo.
>
>
> --
> Esta mensagem foi verificada pelo sistema de antivírus e
> acredita-se estar livre de perigo.

-- 
Esta mensagem foi verificada pelo sistema de antiv�rus e
 acredita-se estar livre de perigo.



Re: [obm-l] |P(z)| > |Q(z)| para uma infinidade de z's

2019-02-11 Por tôpico Bruno Visnadi
Na verdade, é possível que todas aa raízes de P também sejam raízes de Q,
pois não sabemos suas multiplicidade.
Mas haverá uma raiz com multiplicidade maior em P do que em Q, e será
igualmente possível encontrar um r satisfatório.

Em Seg, 11 de fev de 2019 10:47, Claudio Buffara  Suponha que grau(P) = n > m = grau(Q).
> Nesse caso, pela dominância do termo z^n, vai haver R > 0 tal que, pra |z|
> > R, |P(z)| > |Q(z)|.
>
> Por outro lado, como n > m, P tem mais raízes do que Q e, portanto, existe
> a tal que P(a) = 0 e Q(a) <> 0.
> Nesse caso, pela continuidade de P e Q, existe r > 0 tal que |P(z)| <
> |Q(z)| para todo z tal que |z - a| < r.
>
> []s,
> Claudio.
>
>
> On Sun, Feb 10, 2019 at 9:00 PM Artur Steiner <
> artur.costa.stei...@gmail.com> wrote:
>
>> Gostaria de ver a solução dos colegas.
>>
>> Sejam P e Q polinômios complexos não constantes, de graus distintos.
>> Mostre que |P(z)| > |Q(z)| ocorre para uma infinidade de complexos z.
>>
>> Obrigado.
>>
>> Artur Costa Steiner
>>
>> --
>> Esta mensagem foi verificada pelo sistema de antivírus e
>> acredita-se estar livre de perigo.
>
>
> --
> Esta mensagem foi verificada pelo sistema de antivírus e
> acredita-se estar livre de perigo.

-- 
Esta mensagem foi verificada pelo sistema de antiv�rus e
 acredita-se estar livre de perigo.



Re: [obm-l] Teoria de conjuntos

2019-01-27 Por tôpico Bruno Visnadi
Acho que falta alguma informação. Por exemplo, o número total de
matemáticos.

Em Dom, 27 de jan de 2019 09:07, Daniel Quevedo  Dentre matemáticos verificou-se que todos os geômetras eram analistas.
> Metade de todos os analistas eram geômetras. Existem 30 algebristas e 20
> geômetras. Nenhum algebrista é geômetra. O número de analistas que não são
> geômetras nem algebristas é:
>
> R: 5
>
>
> Não consegui chegar na resposta.
> --
> Fiscal: Daniel Quevedo
>
> --
> Esta mensagem foi verificada pelo sistema de antivírus e
> acredita-se estar livre de perigo.

-- 
Esta mensagem foi verificada pelo sistema de antiv�rus e
 acredita-se estar livre de perigo.



Re: [obm-l] OBM 2002 - Problema 6

2019-01-13 Por tôpico Bruno Visnadi
Me parece que o erro está na primeira premissa de que não podemos repetir
as 8 primeiras posições.
A condição do problema é que qualquer par de palavras se difira em 8
posições. Isto é, eles podem ser iguais em até 16 posições.

Em Dom, 13 de jan de 2019 18:11, Pedro José  Boa tarde!
> Suponho ter achado uma solução. Mas pela simplicidade, receio estar
> errada. Fica para ser descartada ou corroborada.
> 1) Vamos primeiro propor que não repitamos as 8 primeiras posições.
> Fixando-se os balores das primeiras 8 posições, tenho 2^16 sequencis de 24
> posições. Das quais só posso aproveitar 1.
> Portanto tenho de descartar 2^16-1 sequencias.
> 2) Como tenho 2^8 possipilidades de escolher as 8 primeiras posições, para
> nunca repertir as 8 primeiras, tenho que descartar ao total:
> 2^8*(2^16-1)=2^24-2^8
> Como tenho 2^24 sequências ao total, só sobrariam 2^8 <4096.
> Como utilizei uma restrição mais branda, para o proposto será menor ainda
> o número de possibilidafes.
> Será aue está correto?
> Saudações,
> PJMS
>
>
>
> Em dom, 13 de jan de 2019 15:37, Pedro José 
>> Jéferson,
>> a sugestão do Cláudio é um caminho.
>> Mas me perdoem-me pela intromissão. Parece que você não percebeu que é um
>> problema de contagem. Você tem 24 casas para preencher com G ou P, mas não
>> pode haver em nenhuma escolha 8 posições preenchidas com os mesmos valores.
>>
>> Em dom, 13 de jan de 2019 14:13, Claudio Buffara <
>> claudio.buff...@gmail.com escreveu:
>>
>>> Tente fazer casos menores, digamos de comprimento 6 ou 8 e diferindo em
>>> pelo menos 2 ou 4 posições.
>>> Deve dar pra fazer na mão (enumeração direta e braçal) e talvez permita
>>> detectar alguma lei de formação.
>>>
>>> On Sat, Jan 12, 2019 at 10:23 PM Jeferson Almir <
>>> jefersonram...@gmail.com> wrote:
>>>
 Amigos peço ajuda nesse problema, e me orientaram a estudar Códigos
 Corretores de Erros.


 Arnaldo e Beatriz se comunicam durante um acampamento usando sinais de
 fumaça, às vezes usando uma nuvem grande, às vezes uma pequena.

 No tempo disponível antes do café da manhã, Arnaldo consegue enviar uma
 seqüência de 24 nuvens. Como Beatriz nem sempre consegue distinguir uma
 nuvem pequena de uma grande, ela e Arnaldo fizeram um dicionário antes de
 ir para o acampamento. No dicionário aparecem N seqüências de 24 tamanhos
 de nuvem (como por exemplo a seqüência PGPGPGPGPGPGGPGPGPGPGPGP, onde G
 significa nuvem grande e P significa nuvem pequena). Para cada uma das N
 seqüências, o dicionário indica seu significado. Para evitar interpretações
 erradas, Arnaldo e Beatriz evitaram incluir no dicionário seqüências
 parecidas. Mais precisamente, duas seqüências no dicionário sempre diferem
 em pelo menos 8 das 24 posições.



 Demonstre que N<= 4096

 --
 Esta mensagem foi verificada pelo sistema de antivírus e
 acredita-se estar livre de perigo.
>>>
>>>
>>> --
>>> Esta mensagem foi verificada pelo sistema de antivírus e
>>> acredita-se estar livre de perigo.
>>
>>
> --
> Esta mensagem foi verificada pelo sistema de antivírus e
> acredita-se estar livre de perigo.

-- 
Esta mensagem foi verificada pelo sistema de antiv�rus e
 acredita-se estar livre de perigo.



Re: [obm-l] Problema

2018-11-26 Por tôpico Bruno Visnadi
Tentei um tabuleiro 12x12 e consegui uma configuração que não tem nenhuma
lâmpada ruim. Acho que dá para estender o padrão para um 2017x2017. Mas me
parece que a paridade importa e talvez o caso 2017x2017 tenha um mínimo de
uma lâmpada ruim.
https://i.imgur.com/HhWrZzu.png

Em seg, 26 de nov de 2018 às 09:27, Claudio Buffara <
claudio.buff...@gmail.com> escreveu:

> Sem pensar muito no problema, aqui vai uma sugestão: tente com um
> tabuleiro menor, 4x4 ou 5x5, pra ver se acha algum padrão.
>
> []s,
> Claudio.
>
> On Mon, Nov 26, 2018 at 9:52 AM  wrote:
>
>> Alguém pode me dar uma sugestão para o problema seguinte?
>>
>> *Problema*
>> Há uma lâmpada em cada casa de um tabuleiro 2019 x 2019 . Cada lâmpada
>> está acesa ou apagada. Uma lâmpada é chamada de *ruim* se ela tem um
>> número par de vizinhas que estão acesas.
>> Qual é o menor número possível de lâmpadas ruins no tabuleiro?
>> * (Duas lâmpadas são vizinhas se elas se encontram em casas do tabuleiro
>> que compartilham um lado.)*
>>
>> NOTA -Tentei raciocinar com o tabuleiro no qual as casas estejam pintadas
>> alternadamente de branco e preto. Desse modo, pode-se ver que como as
>> vizinhas de uma casa branca são todas pretas, parece que uma casa ruim
>> branca não influencia outra branca. Parece que o mesmo deve acontecer com
>> as casas pretas.
>> No caso particular do tabuleiro 3x3, encontrei que o número procurado é 1:
>> B P B
>> P B P
>> B P B
>>
>> Obrigado.
>>
>> Benedito Freire
>>
>>
>> --
>> Esta mensagem foi verificada pelo sistema de antivírus e
>> acredita-se estar livre de perigo.
>>
>
> --
> Esta mensagem foi verificada pelo sistema de antivírus e
> acredita-se estar livre de perigo.

-- 
Esta mensagem foi verificada pelo sistema de antiv�rus e
 acredita-se estar livre de perigo.



Re: [obm-l] Problema 6 - IMO 2001

2018-11-08 Por tôpico Bruno Visnadi
Não entendi. Se a, b, c e d são inteiros, ac e bd certamente são racionais.

Em qui, 8 de nov de 2018 às 22:27, Jeferson Almir 
escreveu:

> Pessoal peço ajuda  no problema :
>
> Sejam a, b , c , d inteiros e a > b > c > d > 0 .
> Suponha que
> ac + bd = ( b+ d + a - c )( b+ d -a + c )
>
> Mostre que ab + cd não é primo .
>
>
> A minha ideia foi:
>
> Abrindo a relação de cima temos
>
> a^2 -ac + c^2 = b^2 + bd + d^2
>
> Então motivado pela ideia de usar geometria que um amigo falou fiz a
> suposição que temos um quadrilátero de lados a, d,b e c respectivamente e
>  nessa ultima relação usando lei dos cossenos teríamos A = 60° e C = 120°
> concluindo então que ABCD é inscritível . Aplicando Ptolomeu temos que ACxBD=
> ab + cd e usando desigualdade triangular podemos afirmar que AC e BD não
> podem ser 1 . Mas ainda tem a possibilidade AC e BD  serem racionais !!
> Como provar que não podem ser ???
>
> --
> Esta mensagem foi verificada pelo sistema de antivírus e
> acredita-se estar livre de perigo.

-- 
Esta mensagem foi verificada pelo sistema de antiv�rus e
 acredita-se estar livre de perigo.



Re: [obm-l] Probabilidade

2018-11-07 Por tôpico Bruno Visnadi
O que o Salhab fez é, na verdade, uma boa cota mínima para esta
probabilidade. Então podemos afirmar que P > 3.16*10^(-15)

Em qua, 7 de nov de 2018 às 17:21, Bruno Visnadi <
brunovisnadida...@gmail.com> escreveu:

> Por que 4*C(46,15)? Talvez seria melhor usar C(46,15)^4 ou, ainda melhor,
> C(46,15)^3, se entendi corretamente a ideia.
>
> Em qua, 7 de nov de 2018 às 16:44, Claudio Buffara <
> claudio.buff...@gmail.com> escreveu:
>
>> Uma desigualdade é:
>> P <= 4*C(46,15) / (60!/(15!)^4) =
>> 4*(15!)^2/(60*59*58*...*48*47*31*30*29*...*17*16) = 7,19336*10^(-22)
>> (se não errei alguma conta...)
>>
>> On Wed, Nov 7, 2018 at 5:24 PM Ralph Teixeira  wrote:
>>
>>> Não tenho a resposta, mas tenho uma boa intuição se for para um contexto
>>> prático: esta probabilidade será super super baixa... :D :D :D
>>>
>>> Uma maneira de estimar é fazer mesmo simulações: faça um programa para
>>> sortear uma ordem, verifique se houve 2 letras iguais adjacentes, repita um
>>> quinquilhão de vezes, veja em quantas deu ou não deu. O problema é que,
>>> como ninha intuição me diz que a probabilidade é baixíssima, eu também
>>> chuto que você vai ter que repetir MUITAS vezes para começa a aparecer
>>> alguma estimativa razoável que não seja 0.
>>>
>>> Abraço, Ralph.
>>>
>>> On Wed, Nov 7, 2018 at 3:28 PM Paulo Rodrigues 
>>> wrote:
>>>
>>>> Muito obrigado pelos avanços.
>>>>
>>>> Se der pra calcular o valor exato melhor, mas se desse pra estimar essa
>>>> probabilidade, eu ficaria satisfeito. Depois explico o contexto prático do
>>>> problema.
>>>>
>>>>
>>>> Paulo Rodrigues
>>>>
>>>>
>>>>
>>>> Em qua, 7 de nov de 2018 às 13:49, Bruno Visnadi <
>>>> brunovisnadida...@gmail.com> escreveu:
>>>>
>>>>> Uma maneira mais simples de colocar os As é imaginar que cada A é uma
>>>>> peça que ocupa 2 espaços, e adicionar um 61º espaço para que seja possível
>>>>> colocar um A na casa 60.
>>>>> Então há 15 As e sobram 61-30 = 31 espaços, e há C(46, 15) maneiras de
>>>>> colocar os As.
>>>>>
>>>>> Em qua, 7 de nov de 2018 às 12:13, Claudio Buffara <
>>>>> claudio.buff...@gmail.com> escreveu:
>>>>>
>>>>>> Fiz mais um pequeno progresso.
>>>>>>
>>>>>> Resolvi um sub-problema.
>>>>>> De quantas formas é possível colocar 15 As nas 60 posições de modo
>>>>>> que 2 As não ocupem posições adjacentes.
>>>>>>
>>>>>> Há 4 casos (exaustivos e mutuamente exclusivos) a considerar:
>>>>>> 1) A primeira e a última posição são ocupadas por As:
>>>>>> Nesse caso, uma vez colocados todos os As, sobrarão, entre eles, 14
>>>>>> "espaços" com comprimentos variados.
>>>>>> Chamando de x(k) o comprimento do k-ésimo espaço, teremos as
>>>>>> condições:
>>>>>> x(k) >= 1, para 1 <= k <= 14.
>>>>>> e
>>>>>> x(1) + x(2) + ... + x(14) = 45  (*)
>>>>>> Logo, o número de maneiras de colocar os As neste caso é igual ao
>>>>>> número de soluções inteiras positivas de (*): C(44,13)
>>>>>>
>>>>>> 2) Um A ocupa a primeira posição mas a última posição está vazia.
>>>>>> A equação, neste caso, é:
>>>>>> x(1) + x(2) + ... + x(15) = 45  com todos os x(k) >= 1 ==> C(44,14).
>>>>>>
>>>>>> 3) Um A ocupa a última posição mas a primeira está vazia:
>>>>>> Por simetria, C(44,14)
>>>>>>
>>>>>> 4) A primeira e a última posições estão vazias:
>>>>>> A equação é x(1) + ... + x(16) = 45   (x(k) >= 1) ==> C(44,15).
>>>>>>
>>>>>> Logo, o número de maneiras de colocar 15 As em 60 posições de modo
>>>>>> que não fiquem dois As adjacentes é igual a:
>>>>>> C(44,13) + 2*C(44,14) + C(44,15)
>>>>>>
>>>>>> Infelizmente, isso abre um monte de sub-casos chatos pra colocação
>>>>>> dos Bs, de modo que não sei se é um caminho promissor. Provavelmente não.
>>>>>>
>>>>>> []s,
>>>>>> Claudio.
>>>>>>
>>>>>>
>>>>>> On Tue, Nov 6, 2018 at 4:01 PM

Re: [obm-l] Probabilidade

2018-11-07 Por tôpico Bruno Visnadi
Por que 4*C(46,15)? Talvez seria melhor usar C(46,15)^4 ou, ainda melhor,
C(46,15)^3, se entendi corretamente a ideia.

Em qua, 7 de nov de 2018 às 16:44, Claudio Buffara <
claudio.buff...@gmail.com> escreveu:

> Uma desigualdade é:
> P <= 4*C(46,15) / (60!/(15!)^4) =
> 4*(15!)^2/(60*59*58*...*48*47*31*30*29*...*17*16) = 7,19336*10^(-22)
> (se não errei alguma conta...)
>
> On Wed, Nov 7, 2018 at 5:24 PM Ralph Teixeira  wrote:
>
>> Não tenho a resposta, mas tenho uma boa intuição se for para um contexto
>> prático: esta probabilidade será super super baixa... :D :D :D
>>
>> Uma maneira de estimar é fazer mesmo simulações: faça um programa para
>> sortear uma ordem, verifique se houve 2 letras iguais adjacentes, repita um
>> quinquilhão de vezes, veja em quantas deu ou não deu. O problema é que,
>> como ninha intuição me diz que a probabilidade é baixíssima, eu também
>> chuto que você vai ter que repetir MUITAS vezes para começa a aparecer
>> alguma estimativa razoável que não seja 0.
>>
>> Abraço, Ralph.
>>
>> On Wed, Nov 7, 2018 at 3:28 PM Paulo Rodrigues  wrote:
>>
>>> Muito obrigado pelos avanços.
>>>
>>> Se der pra calcular o valor exato melhor, mas se desse pra estimar essa
>>> probabilidade, eu ficaria satisfeito. Depois explico o contexto prático do
>>> problema.
>>>
>>>
>>> Paulo Rodrigues
>>>
>>>
>>>
>>> Em qua, 7 de nov de 2018 às 13:49, Bruno Visnadi <
>>> brunovisnadida...@gmail.com> escreveu:
>>>
>>>> Uma maneira mais simples de colocar os As é imaginar que cada A é uma
>>>> peça que ocupa 2 espaços, e adicionar um 61º espaço para que seja possível
>>>> colocar um A na casa 60.
>>>> Então há 15 As e sobram 61-30 = 31 espaços, e há C(46, 15) maneiras de
>>>> colocar os As.
>>>>
>>>> Em qua, 7 de nov de 2018 às 12:13, Claudio Buffara <
>>>> claudio.buff...@gmail.com> escreveu:
>>>>
>>>>> Fiz mais um pequeno progresso.
>>>>>
>>>>> Resolvi um sub-problema.
>>>>> De quantas formas é possível colocar 15 As nas 60 posições de modo que
>>>>> 2 As não ocupem posições adjacentes.
>>>>>
>>>>> Há 4 casos (exaustivos e mutuamente exclusivos) a considerar:
>>>>> 1) A primeira e a última posição são ocupadas por As:
>>>>> Nesse caso, uma vez colocados todos os As, sobrarão, entre eles, 14
>>>>> "espaços" com comprimentos variados.
>>>>> Chamando de x(k) o comprimento do k-ésimo espaço, teremos as condições:
>>>>> x(k) >= 1, para 1 <= k <= 14.
>>>>> e
>>>>> x(1) + x(2) + ... + x(14) = 45  (*)
>>>>> Logo, o número de maneiras de colocar os As neste caso é igual ao
>>>>> número de soluções inteiras positivas de (*): C(44,13)
>>>>>
>>>>> 2) Um A ocupa a primeira posição mas a última posição está vazia.
>>>>> A equação, neste caso, é:
>>>>> x(1) + x(2) + ... + x(15) = 45  com todos os x(k) >= 1 ==> C(44,14).
>>>>>
>>>>> 3) Um A ocupa a última posição mas a primeira está vazia:
>>>>> Por simetria, C(44,14)
>>>>>
>>>>> 4) A primeira e a última posições estão vazias:
>>>>> A equação é x(1) + ... + x(16) = 45   (x(k) >= 1) ==> C(44,15).
>>>>>
>>>>> Logo, o número de maneiras de colocar 15 As em 60 posições de modo que
>>>>> não fiquem dois As adjacentes é igual a:
>>>>> C(44,13) + 2*C(44,14) + C(44,15)
>>>>>
>>>>> Infelizmente, isso abre um monte de sub-casos chatos pra colocação dos
>>>>> Bs, de modo que não sei se é um caminho promissor. Provavelmente não.
>>>>>
>>>>> []s,
>>>>> Claudio.
>>>>>
>>>>>
>>>>> On Tue, Nov 6, 2018 at 4:01 PM Claudio Buffara <
>>>>> claudio.buff...@gmail.com> wrote:
>>>>>
>>>>>> O número de casos possíveis é C(60,15)*C(45,15)*C(30,15)*C(15,15) =
>>>>>> 60!/(15!)^4
>>>>>> (das 60 posições da sequencia, escolhe 15 para colocar os As; das 45
>>>>>> restantes, escolhe mais 15 pra colocar os Bs; etc...)
>>>>>>
>>>>>> O número de casos favoráveis é mais chatinho.
>>>>>> Eu sugiro olhar prum caso menor pra ver se aparece algum padrão.
>>>>>> Por exemplo, 8 q

Re: [obm-l] Probabilidade

2018-11-07 Por tôpico Bruno Visnadi
Não sei se concordo com essa solução, creio que f(2) seja maior que 432. Se
entendi corretamente, você só contou casos em que nenhuma letra se repete
dentro das 4 primeiras ou das 4 últimas, mas há outras possibilidades, como
ABABCDCD.

Em qua, 7 de nov de 2018 às 15:27, Marcelo Salhab Brogliato <
msbro...@gmail.com> escreveu:

> Olá, Paulo, boa tarde.
>
> Pensei da seguinte forma: tentar uma recursão na quantidade de cada uma
> das letras. Assim, a quantidade de formas de montar um gabarito sem ter
> duas letras consecutivas iguais seria f(15).
> Como a propriedade de não ter letras iguais se aplica para qualquer
> subconjunto do gabarito, podemos quebrar em dois pedaços que nenhum dos
> pedaços terá letras iguais. Então, pensei no seguinte:
>
> f(1) é o número de formas de montar um gabarito escolhendo 1x cada uma das
> letras (A, B, C, D). Logo, f(1) = 4!
> Podemos formar f(2) juntando 2x f(1), mas só temos que evitar que última
> letra do primeiro f(1) seja diferente da primeira letra do segundo f(1).
> Por simetria, f(1) termina f(1)/4 vezes em cada letra. E também f(1) inicia
> f(1)/4 vezes em cada letra.
> Assim, separamos em 4 casos bem parecidos. No primeiro caso, temos f(1)/4
> combinações que terminam com A, e vamos juntar com 3 * f(1)/4 combinações
> que não começam com A. Isso é: (f(1)/4) * (3*f(1)/4). Os outros 3 casos são
> iguais e só temos que somar tudo. Assim, fica:
>
> f(2) = 4 * (f(1)/4) * (3*f(1)/4) = 3/4 * f(1) * f(1) = 3/4 * 4! * 4! = 432
>
> Com os mesmos argumentos, podemos generalizar: f(n+1) = 4 * (f(n)/4) *
> (3*f(1)/4) = 3/4*f(n)*f(1)
>
> f(2) = 3/4 * f(1) * f(1) = 432
> f(3) = 3/4 * f(2) * f(1) = 3/4 * 3/4 * f(1) * f(1) * f(1) = (3/4)^2 *
> f(1)^3
> f(4) = 3/4 * f(3) * f(1) = (3/4)^3 * f(1)^4
>
> Generalizando, f(n) = (3/4)^(n-1) * f(1)^n = 4/3 [ 3/4 * f(1) ]^n
>
> Logo, f(15) = 4/3 * (3/4 * 4!)^15 = 4/3 * 18^15
>
> Agora é só dividir pelo total, que o Claudio Buffara já calculou:
> 60!/(15!)^4.
>
> Assim, a probabilidade seria: 4/3 * 18^15 * (15!)^4 / 60!
>
> Fazendo no computador, fica 3.1611849689983148e-15. Ou eu errei feio, ou é
> bem improvável, hein? Hehe ;)
>
> Abraços,
> Salhab
>
>
> Il giorno mer 7 nov 2018 alle ore 15:28 Paulo Rodrigues 
> ha scritto:
>
>> Muito obrigado pelos avanços.
>>
>> Se der pra calcular o valor exato melhor, mas se desse pra estimar essa
>> probabilidade, eu ficaria satisfeito. Depois explico o contexto prático do
>> problema.
>>
>>
>> Paulo Rodrigues
>>
>>
>>
>> Em qua, 7 de nov de 2018 às 13:49, Bruno Visnadi <
>> brunovisnadida...@gmail.com> escreveu:
>>
>>> Uma maneira mais simples de colocar os As é imaginar que cada A é uma
>>> peça que ocupa 2 espaços, e adicionar um 61º espaço para que seja possível
>>> colocar um A na casa 60.
>>> Então há 15 As e sobram 61-30 = 31 espaços, e há C(46, 15) maneiras de
>>> colocar os As.
>>>
>>> Em qua, 7 de nov de 2018 às 12:13, Claudio Buffara <
>>> claudio.buff...@gmail.com> escreveu:
>>>
>>>> Fiz mais um pequeno progresso.
>>>>
>>>> Resolvi um sub-problema.
>>>> De quantas formas é possível colocar 15 As nas 60 posições de modo que
>>>> 2 As não ocupem posições adjacentes.
>>>>
>>>> Há 4 casos (exaustivos e mutuamente exclusivos) a considerar:
>>>> 1) A primeira e a última posição são ocupadas por As:
>>>> Nesse caso, uma vez colocados todos os As, sobrarão, entre eles, 14
>>>> "espaços" com comprimentos variados.
>>>> Chamando de x(k) o comprimento do k-ésimo espaço, teremos as condições:
>>>> x(k) >= 1, para 1 <= k <= 14.
>>>> e
>>>> x(1) + x(2) + ... + x(14) = 45  (*)
>>>> Logo, o número de maneiras de colocar os As neste caso é igual ao
>>>> número de soluções inteiras positivas de (*): C(44,13)
>>>>
>>>> 2) Um A ocupa a primeira posição mas a última posição está vazia.
>>>> A equação, neste caso, é:
>>>> x(1) + x(2) + ... + x(15) = 45  com todos os x(k) >= 1 ==> C(44,14).
>>>>
>>>> 3) Um A ocupa a última posição mas a primeira está vazia:
>>>> Por simetria, C(44,14)
>>>>
>>>> 4) A primeira e a última posições estão vazias:
>>>> A equação é x(1) + ... + x(16) = 45   (x(k) >= 1) ==> C(44,15).
>>>>
>>>> Logo, o número de maneiras de colocar 15 As em 60 posições de modo que
>>>> não fiquem dois As adjacentes é igual a:
>>>> C(44,13) + 2*C(44,14) + C(44,15)
>>>>
>>>&

Re: [obm-l] Probabilidade

2018-11-07 Por tôpico Bruno Visnadi
Uma maneira mais simples de colocar os As é imaginar que cada A é uma peça
que ocupa 2 espaços, e adicionar um 61º espaço para que seja possível
colocar um A na casa 60.
Então há 15 As e sobram 61-30 = 31 espaços, e há C(46, 15) maneiras de
colocar os As.

Em qua, 7 de nov de 2018 às 12:13, Claudio Buffara <
claudio.buff...@gmail.com> escreveu:

> Fiz mais um pequeno progresso.
>
> Resolvi um sub-problema.
> De quantas formas é possível colocar 15 As nas 60 posições de modo que 2
> As não ocupem posições adjacentes.
>
> Há 4 casos (exaustivos e mutuamente exclusivos) a considerar:
> 1) A primeira e a última posição são ocupadas por As:
> Nesse caso, uma vez colocados todos os As, sobrarão, entre eles, 14
> "espaços" com comprimentos variados.
> Chamando de x(k) o comprimento do k-ésimo espaço, teremos as condições:
> x(k) >= 1, para 1 <= k <= 14.
> e
> x(1) + x(2) + ... + x(14) = 45  (*)
> Logo, o número de maneiras de colocar os As neste caso é igual ao número
> de soluções inteiras positivas de (*): C(44,13)
>
> 2) Um A ocupa a primeira posição mas a última posição está vazia.
> A equação, neste caso, é:
> x(1) + x(2) + ... + x(15) = 45  com todos os x(k) >= 1 ==> C(44,14).
>
> 3) Um A ocupa a última posição mas a primeira está vazia:
> Por simetria, C(44,14)
>
> 4) A primeira e a última posições estão vazias:
> A equação é x(1) + ... + x(16) = 45   (x(k) >= 1) ==> C(44,15).
>
> Logo, o número de maneiras de colocar 15 As em 60 posições de modo que não
> fiquem dois As adjacentes é igual a:
> C(44,13) + 2*C(44,14) + C(44,15)
>
> Infelizmente, isso abre um monte de sub-casos chatos pra colocação dos Bs,
> de modo que não sei se é um caminho promissor. Provavelmente não.
>
> []s,
> Claudio.
>
>
> On Tue, Nov 6, 2018 at 4:01 PM Claudio Buffara 
> wrote:
>
>> O número de casos possíveis é C(60,15)*C(45,15)*C(30,15)*C(15,15) =
>> 60!/(15!)^4
>> (das 60 posições da sequencia, escolhe 15 para colocar os As; das 45
>> restantes, escolhe mais 15 pra colocar os Bs; etc...)
>>
>> O número de casos favoráveis é mais chatinho.
>> Eu sugiro olhar prum caso menor pra ver se aparece algum padrão.
>> Por exemplo, 8 questões, com 2 respostas A, 2 B, 2 C e 2 D.
>> Esse sai por inclusão-exclusão, mas com uma expressão meio feia e que não
>> me parece o melhor caminho pro caso do problema.
>> Talvez dê pra achar alguma recorrência ou função geradora.
>>
>> []s,
>> Claudio.
>>
>>
>>
>> On Tue, Nov 6, 2018 at 1:04 PM Paulo Rodrigues  wrote:
>>
>>> Pessoal, alguém pode dar uma mão na seguinte situação:
>>>
>>> Um gabarito é formado por uma sequência de 60 letras A, B, C e D sendo
>>> 15 de cada tipo.
>>> Qual a probabilidade de não existirem duas letras iguais vizinhas?
>>>
>>> Paulo Rodrigues
>>>
>>>
>>> --
>>> Esta mensagem foi verificada pelo sistema de antivírus e
>>> acredita-se estar livre de perigo.
>>
>>
> --
> Esta mensagem foi verificada pelo sistema de antivírus e
> acredita-se estar livre de perigo.

-- 
Esta mensagem foi verificada pelo sistema de antiv�rus e
 acredita-se estar livre de perigo.



[obm-l] Re: [obm-l] Questão de probabilidade

2018-11-06 Por tôpico Bruno Visnadi
Seja Pa a probabilidade de ocorrência de a. Defina Pb e Pc analogamente.
a = Pa(1-Pb)(1-Pc)
b = Pb(1-Pa)(1-Pc)
c = Pc(1-Pa)(1-Pb)
p = (1-Pa)(1-Pb)(1-Pc)
Queremos achar a razão Pa/Pc
Da equação (a - 2b)p = ab, obtemos:
(1-Pa)(1-Pb)(1-Pc)²(Pa(1-Pb) - 2Pb(1-Pa)) = PaPb(1-Pa)(1-Pb)(1-Pc)²
Pa(1-Pb) - 2Pb(1-Pa)  = PaPb
Pa - 2Pb + PaPb = PaPb
Pa = 2Pb -> Pb = Pa/2
Da equação  (b - 3c)p = 2bc, obtemos:
(1-Pa)²(1-Pb)(1-Pc)(Pb(1-Pc) - 3Pc(1-Pb)) = 2PbPc(1-Pa)²(1-Pb)(1-Pc)
Pb(1-Pc) - 3Pc(1-Pb) = 2PbPc
Pb - 3Pc + 2PcPb = 2PbPc
Pb = 3Pc
Logo: Pa/2 = 3Pc
Pa/Pc = 6





Em ter, 6 de nov de 2018 às 12:43, Vanderlei Nemitz 
escreveu:

> Pessoal, alguém tem um ideia de como resolver a seguinte questão? Já
> tentei muita coisa, sem sucesso.
> Muito obrigado!
>
> Vanderlei
>
> Sejam três eventos independentes A, B e C. A probabilidade de que ocorra
> apenas o evento A é a, apenas o evento B é b e apenas o evento C é c. Seja
> p a probabilidade de que nenhum dos eventos A, B ou C ocorra. Sabendo que
> todas as probabilidades citadas são números no intervalo ]0, 1[ e que p
> satisfaz as equações (a - 2b).p = ab e (b - 3c).p = 2bc, a razão entre a
> probabilidade de ocorrência de A e a probabilidade de ocorrência de C é:
> a) 12
> b) 3
> c) 10
> d) 5
> e) 6
>
> --
> Esta mensagem foi verificada pelo sistema de antivírus e
> acredita-se estar livre de perigo.

-- 
Esta mensagem foi verificada pelo sistema de antiv�rus e
 acredita-se estar livre de perigo.



[obm-l] Re: [obm-l] [Problema] Achar o mínimo do valor absoluto de uma soma complexa

2018-11-03 Por tôpico Bruno Visnadi
Não entendi a pergunta - o que é uma excursão?

Em sáb, 3 de nov de 2018 às 22:18, Jardiel Cunha 
escreveu:

> Olá!
>
>
> Estou trabalhando em um projeto e um problema está me tirando o sono há
> algum tempo. Meu trabalho é na área de engenharia de microondas. A solução
> que eu encontrei até agora, acha soluções mas não satisfatórias... Não
> precisam fazer o problema, queria apenas uma luz em que caminho seguir.
>
>
> [Problema] Dados N pontos em um círculo, estou querendo achar um ponto
> dentro do círculo tal que: para qualquer valor de excursão em graus, eu
> garanta que não existe outro ponto que englobe mais pontos no círculo do
> que ele.
>
>
> Por exemplo: se eu der uma excursão de 80 graus... então eu quero um ponto
> tal que englobe o maior número possível desses N pontos estando ele no
> centro de um arco de 80 graus.
>
>
> Mais um exemplo: tenho 10 pontos. Queria um ponto x tal que ele será o
> centro de todos os arcos com o maior número possível de pontos.
>
>
> Primeira pergunta: isso é possível???
>
> Segunda pergunta: como calcular este ponto?
>
>
> Abs
>
>
>
> 
>  Virus-free.
> www.avast.com
> 
> <#m_2503312673449891629_DAB4FAD8-2DD7-40BB-A1B8-4E2AA1F9FDF2>
>
> --
> Esta mensagem foi verificada pelo sistema de antivírus e
> acredita-se estar livre de perigo.
>

-- 
Esta mensagem foi verificada pelo sistema de antiv�rus e
 acredita-se estar livre de perigo.



Re: [obm-l] Basel Problem

2018-08-16 Por tôpico Bruno Visnadi
Este vídeo apresenta uma prova bem interessante:
https://www.youtube.com/watch?v=d-o3eB9sfls;

Em 16 de agosto de 2018 17:20, Israel Meireles Chrisostomo <
israelmchrisost...@gmail.com> escreveu:

> Obrigado por responder Claudio.eu esqueci de dizer de forma totalmente
> elementar, sem usar o conceito de derivadas e integrais, complexos, ou
> mesmo indução, ou a seja só matemática elementar(apenas limites).O link
> está aqui:
>
>- http://bit.ly/2E0dJjV 
>
>
> está na página 126.
>
> Em 16 de agosto de 2018 16:55, Claudio Buffara 
> escreveu:
>
>> Que link?
>>
>> Se o problema da Basiléia for calcular SOMA(n=1...infinito) 1/n^2, eu
>> diria que sai por série de Fourier.
>> Talvez a série de f(x) = x^2 no intervalo [-pi,pi].
>> Não usa complexos (mas poderia) e nem indução.
>>
>> []s,
>> Claudio.
>>
>>
>> 2018-08-15 16:58 GMT-03:00 Israel Meireles Chrisostomo <
>> israelmchrisost...@gmail.com>:
>>
>>> Olá pessoal, vim trazer um problema bacana que eu elaborei e gostaria
>>> que vcs me ajudassem.O problema pede para resolver o problema da Basileia
>>> sem usar números complexos ou indução.Alguém aí poderia me ajudar?Gostaria
>>> de ver se a solução é semelhante a minha, ou se tem alguma outra mais
>>> elegante.Minha solução se encontra no link para o meu PDF.
>>> Obrigado a todos!!!
>>>
>>> --
>>> Israel Meireles Chrisostomo
>>>
>>> --
>>> Esta mensagem foi verificada pelo sistema de antivírus e
>>> acredita-se estar livre de perigo.
>>
>>
>>
>> --
>> Esta mensagem foi verificada pelo sistema de antivírus e
>> acredita-se estar livre de perigo.
>
>
>
>
> --
> Israel Meireles Chrisostomo
>
> --
> Esta mensagem foi verificada pelo sistema de antivírus e
> acredita-se estar livre de perigo.

-- 
Esta mensagem foi verificada pelo sistema de antiv�rus e
 acredita-se estar livre de perigo.



[obm-l] Re: [obm-l] Re: [obm-l] Re: [obm-l] Polinômio com raízes reais

2018-07-04 Por tôpico Bruno Visnadi
Opa, sim, quis dizer relativo.

Em 4 de julho de 2018 23:54, Claudio Buffara 
escreveu:

> Ou, melhor dizendo, mínimo ou máximo local.
>
> 2018-07-04 23:52 GMT-03:00 Claudio Buffara :
>
>> Você quer dizer mínimo ou máximo relativo, certo?
>>
>> 2018-07-04 23:42 GMT-03:00 Bruno Visnadi :
>>
>>> Se todas as raízes forem distintas, é possível visualizar isto
>>> geometricamente. Imaginando o gráfico de P, entre quaisquer duas raízes
>>> consecutivas deve haver um máximo absoluto ou um mínimo absoluto de P, e
>>> portanto, uma raiz de P'.
>>>
>>>
>>> Em 4 de julho de 2018 23:17, Artur Steiner <
>>> artur.costa.stei...@gmail.com> escreveu:
>>>
>>>> Acho um tanto surpreendente que este fato não pareça ser muito
>>>> conhecido:
>>>>
>>>> Se todas as raízes de um polinômio P de grau >= 2 forem reais, então
>>>> todas as raízes de P' também são.
>>>>
>>>> Isso vale inclusive para polinômios complexos. Mas basta provar para
>>>> polinômios com coeficientes reais.
>>>>
>>>>
>>>> Artur Costa Steiner
>>>>
>>>> --
>>>> Esta mensagem foi verificada pelo sistema de antivírus e
>>>> acredita-se estar livre de perigo.
>>>
>>>
>>>
>>> --
>>> Esta mensagem foi verificada pelo sistema de antivírus e
>>> acredita-se estar livre de perigo.
>>>
>>
>>
>
> --
> Esta mensagem foi verificada pelo sistema de antivírus e
> acredita-se estar livre de perigo.
>

-- 
Esta mensagem foi verificada pelo sistema de antiv�rus e
 acredita-se estar livre de perigo.



[obm-l] Re: [obm-l] Polinômio com raízes reais

2018-07-04 Por tôpico Bruno Visnadi
Se todas as raízes forem distintas, é possível visualizar isto
geometricamente. Imaginando o gráfico de P, entre quaisquer duas raízes
consecutivas deve haver um máximo absoluto ou um mínimo absoluto de P, e
portanto, uma raiz de P'.


Em 4 de julho de 2018 23:17, Artur Steiner 
escreveu:

> Acho um tanto surpreendente que este fato não pareça ser muito conhecido:
>
> Se todas as raízes de um polinômio P de grau >= 2 forem reais, então todas
> as raízes de P' também são.
>
> Isso vale inclusive para polinômios complexos. Mas basta provar para
> polinômios com coeficientes reais.
>
>
> Artur Costa Steiner
>
> --
> Esta mensagem foi verificada pelo sistema de antivírus e
> acredita-se estar livre de perigo.

-- 
Esta mensagem foi verificada pelo sistema de antiv�rus e
 acredita-se estar livre de perigo.



[obm-l] Re: [obm-l] Re: [obm-l] Re: [obm-l] Re: [obm-l] Números primos

2018-06-09 Por tôpico Bruno Visnadi
15^(4k + 3) = 98 (mod 113), para todo k inteiro. E 15^15 = 3 (mod 4)
Então, 15^(15^15) + 15 = 98 + 15 = 0 mod (113), isto é, 113 divide
15^(15^15) + 15.

Em 9 de junho de 2018 15:55, Pedro José  escreveu:

> Boa tarde!
> Alguém poderia dizer se 113 divide ou não 15^(15^15) +15?
>
> Saudações,
> PJMS
>
>
> Em Sex, 8 de jun de 2018 15:41, Pedro José  escreveu:
>
>> Boa tarde!
>>
>> Ajudem-me.
>> p=113 ==> Fi(113) = 112
>>
>> 15^(15^15) = 15^b onde b = 15^15 mod 112.
>> 15^15= 15 mod 112.
>> 15^(15^15)= 15^(k.112+15)= (15^112)^k*15^15=15^15 mod 113
>> 15^(15^15-1)= 15^14= -1 mod 13
>> logo 113 também divide 15^(15^15) + 15.
>> 113 é primo.
>> O enunciado deveria ser dos 4 menores fatores primos de...
>>
>> Ou está errado que 113 | 15^(15^15)+15
>>
>> Saudações,
>> PJMS
>>
>>
>> Em 8 de junho de 2018 15:27, Pedro José  escreveu:
>>
>>> Boa tarde!
>>> Já tinha corrigido.
>>> Mas não consigo vislumbrar, por que só existem esses 4 primos: 2, 3, 5 e
>>> 29.
>>>
>>> Em 8 de junho de 2018 14:24, Otávio Araújo 
>>> escreveu:
>>>
 O número 15^(15^15 - 1) + 1 é par, logo não pode ser da forma 29^k

 Em sex, 8 de jun de 2018 2:21 PM, Pedro José 
 escreveu:

> Boa tarde!
> Não tive tempo de corrigir.
> Seja a= 15^15
> p | 15(15^(a-1) +1); Não subtrai 1 de 15^15, na primeira feita, quando
> coloquei 15 em evidência.
>
> p<>3 e p<>5 ==> 15^(a-1) = -1 mod p
> p=7 ==> 15^(a-1) = 1; p=7 não atende.
> b=a mod(p-1) ==> 15^(a-1)=15^(b-1) mod p
> p=11 ==> b= a = 5 mod 10 15^a= 15^5 mod11
> 15^(a-1)=15^4= 3 mod11. p=11 não atende.
> p=13 ==> b= 15^15=3 mod 12 ==> 15^(a-1)=15^2= 4 mod13; p=13 não atende.
> p=17 ==> b= 15^15 = 15 mod 16 ==> 15(a-1)=15^14<>-1 mod17, pois, 15^4
> = -1 e 4 não divide 14; p=17 não atende.
> p=19 ==> b= 15^15=9 mod18 ==> 15^(a-1) = 15^8 = 5 mod 19; p=19 não
> atende
> p=23 ==> b= 15^15=1 mod22 ==> 15(a-1) = 1 mod 23; p=23 não atende
> p=29 ==> b= 15^15 = 15 mod 28 ==>15^(a-1) = 15^14= -1 mod29.
>
> O outro primo é 29.
>
> Porém, se não há a dica que só tem mais um fator primo, boiaria.
> Agora, o objetivo é procurar uma forma de mostrar que 15^(15^15 - 1) + 1 =
> 29^k, com k natural.
>
> Saudações,
> PJMS.
>
> Em 7 de junho de 2018 23:31, Pedro José 
> escreveu:
>
>> Boa noite.
>> Desconsiderar.
>> Está errado.
>>
>> Em Qui, 7 de jun de 2018 23:10, Pedro José 
>> escreveu:
>>
>>> Boa noite!
>>> p| 15(15^(15^15)+1) então:
>>> 15^(15^15) = -1 mod p.
>>>
>>> Como 15^(p-1) =1 mod p
>>> 15^(15^15) = 15^a, onde a=15^15 mod(p-1).
>>> Como o problema da a dica de que são apenas 4 primos.isso não pensei
>>> como mostrar, sem a dica do enunciado.
>>> Aí, você começa com p=7 e continua até achar o primo desejado.
>>> Para p=7 da de cara:15^(15^15)=1 mod7, não atende.
>>> Para p=11, 15^15=5 mod10
>>> 15^(15^15)=15^5=1 mod 11, não atende.
>>> Até chegar a p=31.
>>> 15^15= 15 mod 30
>>> 15^15 = ? mod 31
>>> 15^2=8 mod 31
>>> 15^4 =64=2 mod 31
>>> 14^8=4 mod 31
>>> 15^14=8*2*4=2 mod  31.
>>> 15^15= -1 mod 31.
>>> Então o outro primo é 31.
>>> Saudações,
>>> PJMS.
>>>
>>> Em Qui, 7 de jun de 2018 18:27, Daniel Quevedo 
>>> escreveu:
>>>
 A soma dos 4 fatores primos distintos do número 15^(15^15) + 15 é:
 R: 39

 Pergunta: dá pra saber rápido q se colocarmos 15 em evidência temos
 os fatores 3 e 5. Como a soma de dois ímpares é sempre par, o 2 tbm é 
 fator.
 Minha dificuldade é descobrir o terceiro
 --
 Fiscal: Daniel Quevedo

 --
 Esta mensagem foi verificada pelo sistema de antivírus e
 acredita-se estar livre de perigo.
>>>
>>>
>
> --
> Esta mensagem foi verificada pelo sistema de antivírus e
> acredita-se estar livre de perigo.


 --
 Esta mensagem foi verificada pelo sistema de antivírus e
 acredita-se estar livre de perigo.

>>>
>>>
>>
> --
> Esta mensagem foi verificada pelo sistema de antivírus e
> acredita-se estar livre de perigo.

-- 
Esta mensagem foi verificada pelo sistema de antiv�rus e
 acredita-se estar livre de perigo.



[obm-l] Re: [obm-l] Re: [obm-l] Re: [obm-l] Função Composta

2018-05-13 Por tôpico Bruno Visnadi
Realmente eu me expressei mal ali. Eu quis dizer que o menor N deve ser 1,
2 ou 5.

Em 13 de maio de 2018 21:22, Jeferson Almir <jefersonram...@gmail.com>
escreveu:

> Boa noite.
> Eu só não entendi essa passagem
>  “ Para todo a, queremos que N seja igual a 1, 2 ou 5 (os divisores de 50
> menores ou iguais a 5).“
> Pois pra mim eu teria que levar em conta somente os divisores de 50
>
> Em dom, 13 de mai de 2018 às 19:43, Bruno Visnadi <
> brunovisnadida...@gmail.com> escreveu:
>
>> Não sei se ficou meio confuso:
>> De fato a função é injetiva, pois se f(a) = f(b) então f^50(a) = f^50(b)
>> e a = b. E claramente é sobrejetiva, portanto, é bijetiva. Existem 5! = 120
>> bijeções de S em S. Vamos descontar as que não tem a propriedade desejada.
>> Em cada bijeção de S em S, dado um a, existe um menor N tal que f^N(a) =
>> a. Para todo a, queremos que N seja igual a 1, 2 ou 5 (os divisores de 50
>> menores ou iguais a 5).
>> Se existem um a cujo N é igual a 3, temos um caso em que f(a) = b, f(b)
>> = c e f(c) = a . Existem 10 maneiras de escolher a, b, c de S, duas
>> maneiras de escolher o 'ciclo' entre eles (a->b->c ou a->c->b), e mais 2
>> maneiras de escolher a imagem dos outros 2 elementos (se forem x e y,
>> podemos ter f(x) = x e f(y) = y ou f(x) = y e f(y) = x). Então temos 10*2*2
>> = 40 funções deste tipo.
>> Se existe um a cujo N é igual a 4, temos um caso em que f(a) = b, f(b) =
>> c e f(c) = d e f(d) = a. Temos 5 maneiras de escolher estes 4 elementos de
>> S, e mais 6 maneiras de ordenar o 'ciclo' entre eles (basta fixar um deles
>> e vemos que são 3! maneiras). Então 6*5 = 30 funções deste tipo.
>> Logo a quantidade de funções com as propriedades que buscamos é 120-40-30
>> = 50.
>>
>> Em 13 de maio de 2018 18:03, Jeferson Almir <jefersonram...@gmail.com>
>> escreveu:
>>
>>> Seja S = { 1,2,3,4,5 }, quantas são as funções de f: S -> S tais que
>>> f^50(x)= x para todo x pertencente a S ?? ( f^50(x) = fofofo...of(x)
>>> Eu provei que ela era injetiva e acho que provei também que ela era
>>> sobrejetiva mas minha resposta dar 45 . O gabarito diz que são 50. Desde já
>>> agradeço qualquer ajuda.
>>>
>>> --
>>> Esta mensagem foi verificada pelo sistema de antivírus e
>>> acredita-se estar livre de perigo.
>>
>>
>> --
>> Esta mensagem foi verificada pelo sistema de antivírus e
>> acredita-se estar livre de perigo.
>
>
> --
> Esta mensagem foi verificada pelo sistema de antivírus e
> acredita-se estar livre de perigo.
>

-- 
Esta mensagem foi verificada pelo sistema de antiv�rus e
 acredita-se estar livre de perigo.



[obm-l] Re: [obm-l] Função Composta

2018-05-13 Por tôpico Bruno Visnadi
Não sei se ficou meio confuso:
De fato a função é injetiva, pois se f(a) = f(b) então f^50(a) = f^50(b) e
a = b. E claramente é sobrejetiva, portanto, é bijetiva. Existem 5! = 120
bijeções de S em S. Vamos descontar as que não tem a propriedade desejada.
Em cada bijeção de S em S, dado um a, existe um menor N tal que f^N(a) = a.
Para todo a, queremos que N seja igual a 1, 2 ou 5 (os divisores de 50
menores ou iguais a 5).
Se existem um a cujo N é igual a 3, temos um caso em que f(a) = b, f(b) = c
e f(c) = a . Existem 10 maneiras de escolher a, b, c de S, duas maneiras de
escolher o 'ciclo' entre eles (a->b->c ou a->c->b), e mais 2 maneiras de
escolher a imagem dos outros 2 elementos (se forem x e y, podemos ter f(x)
= x e f(y) = y ou f(x) = y e f(y) = x). Então temos 10*2*2 = 40 funções
deste tipo.
Se existe um a cujo N é igual a 4, temos um caso em que f(a) = b, f(b) = c
e f(c) = d e f(d) = a. Temos 5 maneiras de escolher estes 4 elementos de S,
e mais 6 maneiras de ordenar o 'ciclo' entre eles (basta fixar um deles e
vemos que são 3! maneiras). Então 6*5 = 30 funções deste tipo.
Logo a quantidade de funções com as propriedades que buscamos é 120-40-30 =
50.

Em 13 de maio de 2018 18:03, Jeferson Almir 
escreveu:

> Seja S = { 1,2,3,4,5 }, quantas são as funções de f: S -> S tais que
> f^50(x)= x para todo x pertencente a S ?? ( f^50(x) = fofofo...of(x)
> Eu provei que ela era injetiva e acho que provei também que ela era
> sobrejetiva mas minha resposta dar 45 . O gabarito diz que são 50. Desde já
> agradeço qualquer ajuda.
>
> --
> Esta mensagem foi verificada pelo sistema de antivírus e
> acredita-se estar livre de perigo.

-- 
Esta mensagem foi verificada pelo sistema de antiv�rus e
 acredita-se estar livre de perigo.



[obm-l] Re: [obm-l] Re: [obm-l] Re: [obm-l] Função Composta

2018-05-11 Por tôpico Bruno Visnadi
 Vou considerar que 0 é natural (para N = {1, 2, 3...} a prova é análoga).
Lema 1: f é injetora.
Prova: Se f(a) = f(b) então f(f(a)) = f(f(b)) e a = b.
Lema 2: Se f(a) > 2004, então a está na imagem de f.
Prova: Se f(a) > 2004, então f(f(f(a) - 2005)) = f(a). Como a função é
injetora, f(f(a) - 2005) = a.
Suponha que existe uma função f, N -> N, tal que f(f(n)) = n + 2005.
Seja S o conjunto dos 2005 naturais [0, 2004]. Suponha que existam 1003
elementos t de S tais que f(t) ∈ S. Portanto, há no máximo 1002 elementos t
de S tais que f(t) ∉ S. Assim, uma vez que a função é injetora, pelo
princípio das casas dos pombos haveria um elemento t tal que f(f(t)) ∈ S ⇒
2015 + t  ∈ S, absurdo, pois 2005 + t > 2004.

Então existem pelo menos 1003 elementos t de S com f(t) > 2004. Sejam a1,
a2 (...) a1003 tais elementos. Pelo Lema 2, estes números estão na imagem
de f. Então existem 1003 números b1, b2, (...) b1003 tais que f(b1) = a1,
f(b2) = a2, (...) f(b1003) = a1003. Não podem haver i e j tais que bi = aj,
pois f(bi) < 2005 e f(aj) > 2004. Assim, se bi ∈ S, bi é um dos 1002
elementos t de S com f(t) ∈ S. Mas existem 1003 números bi, portanto, ao
menos um deles não pertence a S. Seja bk tal elemento, com f(bk) = ak. Pelo
Lema 2, bk está na imagem de f, então existe c com f(c) = bk ⇒ f(f(c)) =
f(bk) ⇒ 2005 + c = ak. Absurdo, pois ak < 2005.

Portanto, não existe tal f.

Em 11 de maio de 2018 18:46, Rodrigo Ângelo 
escreveu:

> acho que, de forma mais geral, não pode existir nenhuma f: |N -> |N, tal
> que f(f(n)) = n*p(n) + i, onde g(n) seja qualquer polinômio natural de n e
> i é um número ímpar
>
> On Fri, May 11, 2018 at 6:37 PM Rodrigo Ângelo 
> wrote:
>
>> Se f não for polinomial, então f deve ser da forma f(n) = g(n) + m, onde
>> g(n) é uma função não polinomial de n e m é um natural ou zero
>> f(f(n)) = g(f(n)) + m
>>
>> Com f(f(n)) = n + 2005, teríamos
>> g(f(n)) + m = n  + 2005
>> g(f(n)) = n  + 2005 - m onde m é uma constante natural então g(f(n)) é um
>> polinômio, que é um absurdo.
>>
>> On Fri, May 11, 2018 at 6:20 PM Rodrigo Ângelo 
>> wrote:
>>
>>> Se f for qualquer polinômio de grau maior que 1 então f(f(n)) também é
>>> um polinomio maior que 1. Daí já dá pra eliminar toda f polinomial
>>>
>>> On Fri, May 11, 2018 at 6:15 PM Julio César Saldaña Pumarica <
>>> saldana...@pucp.edu.pe> wrote:
>>>
 com isso prova que f nao pode ser linear mas o enunciado pareces mais
 geral

 El viernes, 11 de mayo de 2018, Rodrigo Ângelo 
 escribió:

> Se f : |N -> |N, f(n) = an + m, com a e m constantes naturais, então
> teríamos
> f(f(n)) = a(an + m) + m
> f(f(n)) = (a^2)n + am + m
>
> Com f(f(n)) = n + 2005, teríamos a = 1 e m = 2005/2, absurdo, pois m
> deve ser um número natural.
>
> On Fri, May 11, 2018 at 10:51 AM Jeferson Almir <
> jefersonram...@gmail.com> wrote:
>
>> Como provar que nos naturais não existe a função f ( f(n) ) = n +
>> 2005 ???
>>
>> --
>> Esta mensagem foi verificada pelo sistema de antivírus e
>> acredita-se estar livre de perigo.
>
>
> --
> Esta mensagem foi verificada pelo sistema de antivírus e
> acredita-se estar livre de perigo.


 --
 Esta mensagem foi verificada pelo sistema de antivírus e
 acredita-se estar livre de perigo.
>>>
>>>
> --
> Esta mensagem foi verificada pelo sistema de antivírus e
> acredita-se estar livre de perigo.
>

-- 
Esta mensagem foi verificada pelo sistema de antiv�rus e
 acredita-se estar livre de perigo.



[obm-l] Re: [obm-l] Re: [obm-l] Função Composta

2018-05-11 Por tôpico Bruno Visnadi
Acredito que isso só prova que a função não pode ser um polinômio do
primeiro grau, mas não prova que ela não existe.

Em 11 de maio de 2018 17:21, Rodrigo Ângelo 
escreveu:

> Se f : |N -> |N, f(n) = an + m, com a e m constantes naturais, então
> teríamos
> f(f(n)) = a(an + m) + m
> f(f(n)) = (a^2)n + am + m
>
> Com f(f(n)) = n + 2005, teríamos a = 1 e m = 2005/2, absurdo, pois m deve
> ser um número natural.
>
> On Fri, May 11, 2018 at 10:51 AM Jeferson Almir 
> wrote:
>
>> Como provar que nos naturais não existe a função f ( f(n) ) = n + 2005 ???
>>
>> --
>> Esta mensagem foi verificada pelo sistema de antivírus e
>> acredita-se estar livre de perigo.
>
>
> --
> Esta mensagem foi verificada pelo sistema de antivírus e
> acredita-se estar livre de perigo.

-- 
Esta mensagem foi verificada pelo sistema de antiv�rus e
 acredita-se estar livre de perigo.



[obm-l] Re: [obm-l] Como minorar a interseção de 2 conjuntos específicos num diagrama com 3 conjuntos.

2018-02-25 Por tôpico Bruno Visnadi
Acredito que a intersecção mínima seja  #(A) + #(B) - #(total). No caso,
90+80-100 = 70, ou seja, pelo menos 70 pessoas possuem as doenças A e B.

Em um grupo de M pessoas doentes, sendo A1, A2... AN as doenças, a
intersecção mínima das doenças Ak1, Ak2, Ak3, (...), Akt é #(Ak1) + #(Ak2)
+ #(Ak3), (...), + #(Akt) - (t-1)M.
Claro, se este valor for negativo, então a intersecção mínima é
simplesmente 0.

Em 25 de fevereiro de 2018 13:20, Thiago Póvoa 
escreveu:

> Bom Dia.
>
>
> Encontrei uma questão aparentemente fácil, mas que não consegui uma
> solução geral.
>
> Dados 3 conjuntos A, B e C, conhecemos #(A), #(B), #(C) e #(A ou B ou C).
> Como encontrar um limitante inferior para alguma interseção dupla, por
> exemplo, #(A e B)?
>
> Um exemplo de problema com dados numéricos que tirei do livro "Problemas
> Selecionados de Matemática VOL I": Num grupo de 100 pessoas, 90 possuem a
> doença A, 80 a doença B e 70 a doença C. Quantas pessoas, no mínimo,
> possuem as doenças A e B?
>
> Eu não consegui utilizar a desigualdade de Bonferroni, pois ela só me dá
> um valor mínimo para #(A e B e C). Também tentei utilizar o Princípio da
> Inclusão-Exclusão, mas não consegui concluir nada efetivo, pois ele só me
> permite trabalhar com a soma das 3 interseções 2 a 2, e não com cada uma
> delas individualmente.
>
> Enfim, agradeço se alguém tiver alguma ideia para uma solução geral.
>
>
>
> Abraços,
> Thiago Póvoa.
>
> --
> Esta mensagem foi verificada pelo sistema de antivírus e
> acredita-se estar livre de perigo.

-- 
Esta mensagem foi verificada pelo sistema de antiv�rus e
 acredita-se estar livre de perigo.



Re: [obm-l] Probabilidade

2017-12-06 Por tôpico Bruno Visnadi
De fato, eu fiz uma bobagem no wolframalpha. Agora cheguei nos mesmos
0.21881112621423598
do Nowras

Em 6 de dezembro de 2017 21:07, Nowras Ali  escreveu:

> Caro Douglas,
>
> Acredito que a probabilidade seria P = (\binom{90}{1} \binom{110}{1}
> \binom{80}{1})/\binom{280}{3} = (110
> *90*80)/\binom{280}{3}
> = 0.21881112621423598.
>
> Abraços,
> Nowras.
>
> Em 6 de dezembro de 2017 19:58, Douglas Oliveira de Lima <
> profdouglaso.del...@gmail.com> escreveu:
>
>> Caros amigos, preciso da ajuda dos senhores para confirmar um gabarito de
>> uma questão:
>>
>> Eis a questão:
>>
>> Num trem existem 280 animais, sendo 90 da fazenda Tampa, 110 da fazenda
>> Boa Vista, e 80 da fazenda Monte verde, se três dos animais fossem
>> escolhidos ao acaso entre os 280, qual a probabilidade de que cada um deles
>> seja de uma fazenda diferente?
>>
>> Douglas Oliveira.
>>
>> --
>> Esta mensagem foi verificada pelo sistema de antivírus e
>> acredita-se estar livre de perigo.
>
>
>
> --
> Esta mensagem foi verificada pelo sistema de antivírus e
> acredita-se estar livre de perigo.
>

-- 
Esta mensagem foi verificada pelo sistema de antiv�rus e
 acredita-se estar livre de perigo.



Re: [obm-l] Probabilidade

2017-12-06 Por tôpico Bruno Visnadi
Talvez eu tenha feito alguma bobagem, mas cheguei em uma resposta
estranha: 114943/542934

Em 6 de dezembro de 2017 19:58, Douglas Oliveira de Lima <
profdouglaso.del...@gmail.com> escreveu:

> Caros amigos, preciso da ajuda dos senhores para confirmar um gabarito de
> uma questão:
>
> Eis a questão:
>
> Num trem existem 280 animais, sendo 90 da fazenda Tampa, 110 da fazenda
> Boa Vista, e 80 da fazenda Monte verde, se três dos animais fossem
> escolhidos ao acaso entre os 280, qual a probabilidade de que cada um deles
> seja de uma fazenda diferente?
>
> Douglas Oliveira.
>
> --
> Esta mensagem foi verificada pelo sistema de antivírus e
> acredita-se estar livre de perigo.

-- 
Esta mensagem foi verificada pelo sistema de antiv�rus e
 acredita-se estar livre de perigo.



[obm-l] Re: [obm-l] polinômios

2017-11-27 Por tôpico Bruno Visnadi
As raízes precisam ser distintas? Se podem ser iguais, x^4 - 3 x^3 + 3x^2 -
1x é um contra-exemplo ao problema.

Em 27 de novembro de 2017 20:09, André Lauer 
escreveu:

> Boa noite, preciso de ajuda no seguinte problema:
> Um polinômio P(x) tem coeficientes inteiros e admite quatro raízes
> inteiras. Prove que a equação P(x) = 2 não admite raízes inteiras.
>
> --
> Esta mensagem foi verificada pelo sistema de antivírus e
> acredita-se estar livre de perigo.
>

-- 
Esta mensagem foi verificada pelo sistema de antiv�rus e
 acredita-se estar livre de perigo.



[obm-l] Re: [obm-l] Lógica

2017-11-26 Por tôpico Bruno Visnadi
"Mas se uma proposição é falsa, então sua contra-positiva também é falsa"
Sim, e a afirmação '' se y é racional então x é irracional'' é de fato
falsa. Por exemplo, ''Todo ser humano é um réptil'' é uma afirmação falsa.
Sua contrapositiva, ''Se algo não é réptil, então não é humano'', é falsa
também, pois humanos não são répteis e são humanos. É claro, leões não são
répteis e nem humanos, mas isso não torna a sentença verdadeira. O leão é
equivalente ao x = 1 + sqrt(2).

Em 26 de novembro de 2017 20:28, Israel Meireles Chrisostomo <
israelmchrisost...@gmail.com> escreveu:

>
> Seja y=x-1/x.A proposição: se x é racional então y é irracional, é uma
> proposição claramente falsa.Mas se uma proposição é falsa, então sua
> contra-positiva também é falsa.A contra-positiva dessa sentença é:  se y é
> racional então x é irracional, e por ser a proposição contra-positiva da
> primeira, então essa sentença também é falsa.Mas se essa sentença é falsa,
> então se y é racional então x só pode ser racional.Agora veja que se
> x=1+sqrt{2}, podemos ver claramente que y será racional, ou seja, o fato de
> y ser racional não implicaria que x é racional.O que eu fiz de errado?
> --
> Israel Meireles Chrisostomo
>
> --
> Esta mensagem foi verificada pelo sistema de antivírus e
> acredita-se estar livre de perigo.

-- 
Esta mensagem foi verificada pelo sistema de antiv�rus e
 acredita-se estar livre de perigo.



[obm-l] Re: [obm-l] Re: [obm-l] Números triangulares

2017-08-09 Por tôpico Bruno Visnadi
Ainda assim, todo número natural ímpar é a diferença de dois números
triangulares não consecutivos. O problema é uma 'pegadinha', mesmo!

Em 9 de agosto de 2017 22:40, Israel Meireles Chrisostomo <
israelmchrisost...@gmail.com> escreveu:

> Se ele tivesse dito triangulares não consecutivos, aí talvez o problema
> ficaria mais interessante.
>
> Em 9 de agosto de 2017 22:32, Israel Meireles Chrisostomo <
> israelmchrisost...@gmail.com> escreveu:
>
>> Esse problema foi formulado de modo a enganar o leitor, ao se colocar
>> muitos detalhes que nos confundem.Talvez o autor do problema tenha
>> encontrado uma relação mais complexa, mas como o problema está muito
>> abrangente, o problema se resolve facilmente por essa observação.
>>
>> Em 9 de agosto de 2017 22:19, Israel Meireles Chrisostomo <
>> israelmchrisost...@gmail.com> escreveu:
>>
>>> A diferença t(n+1)-t(n)=(n+1)(n+2)/2-n(n+1)/2=n+1  qualquer número
>>> natural maior do que 0 é a diferença de dois números triangulares
>>>
>>> Em 9 de agosto de 2017 21:23, Pedro Chaves 
>>> escreveu:
>>>
 Caros Colegas,
 Seja N = {1, 2, 3, 4, 5, ...} o conjunto dos números naturais.
 Chamamos de número triangular a qualquer número obtido pela expressão
  t(n) = n.(n+1) / 2, sendo n um natural qualquer.
 Como podemos provar que o quadrado de qualquer número natural ímpar,
 múltiplo de 3, é a diferença entre dois números triangulares?
 Abraços do Pedro Chaves.
 
 ---




 --
 Esta mensagem foi verificada pelo sistema de antivírus e
 acredita-se estar livre de perigo.

>>>
>>>
>>>
>>> --
>>> Israel Meireles Chrisostomo
>>>
>>
>>
>>
>> --
>> Israel Meireles Chrisostomo
>>
>
>
>
> --
> Israel Meireles Chrisostomo
>
> --
> Esta mensagem foi verificada pelo sistema de antivírus e
> acredita-se estar livre de perigo.

-- 
Esta mensagem foi verificada pelo sistema de antiv�rus e
 acredita-se estar livre de perigo.



[obm-l] Re: [obm-l] Re: [obm-l] Re: [obm-l] Polinômios

2017-07-25 Por tôpico Bruno Visnadi
Os polinômios que mencionei no formato Q(x) + nP(x) não são necessariamente
múltiplos de (x-1). Mas Q(x) é um exemplo de polinômio que estamos
procurando.

Pelo o que entendi, dois polinômios diferentes podem deixar restos
diferentes, desde que seja o mesmo resto para (x-1), (x-2) e (x-3), certo?

Neste caso, basta tormarmos Qm(x) = m*P(x) + 6m, para todo m. Cada
polinômio deixará resto 6t por (x-1), (x-2) e (x-3).

Qm(x) = mx³ - 9mx² + 26mx - 12m -> Qm(1) = 0. Então, dessa vez eles são
todos múltiplos de (x-1) :)



Em 25 de julho de 2017 22:13, Bruno Visnadi <brunovisnadida...@gmail.com>
escreveu:

> Opa, deixei passar um erro bem básico! Estou corrigindo, um momentinho
>
> Em 25 de julho de 2017 22:04, Pedro Júnior <pedromatematic...@gmail.com>
> escreveu:
>
>> Obrigado, didático e criativo.
>> Valeu mesmo!
>>
>> Em 25 de jul de 2017 9:55 PM, "Bruno Visnadi" <
>> brunovisnadida...@gmail.com> escreveu:
>>
>>> Seja P(x) = (x-2)(x-3)(x-4) = x³ - 9x² + 26x - 24 -> P(1) = -6
>>>
>>> Seja Q(x) = P(x) + 6 -> Q(1) = 0 -> Q(x) é múltiplo de (x-1)
>>>
>>> Perceba que Q(x) deixa resto 6 por (x-2), (x-3) e (x-4). Todo polinômio
>>> no formato Q(x) + n*P(x), para todo n, deixa resto 6 por
>>> (x-2), (x-3) e (x-4).
>>>
>>> Em 25 de julho de 2017 21:22, Pedro Júnior <pedromatematic...@gmail.com>
>>> escreveu:
>>>
>>>> Prove que existem infinitos polinômios de grau 3 de coeficientes reais
>>>> que são divisíveis por x - 1 e que deixam o mesmo resto por x - 2, x - 3 e
>>>> x - 4.
>>>>
>>>> Quem tiver uma boa dica fica meus agradecimentos.
>>>>
>>>> --
>>>> Esta mensagem foi verificada pelo sistema de antivírus e
>>>> acredita-se estar livre de perigo.
>>>
>>>
>>>
>>> --
>>> Esta mensagem foi verificada pelo sistema de antivírus e
>>> acredita-se estar livre de perigo.
>>
>>
>> --
>> Esta mensagem foi verificada pelo sistema de antivírus e
>> acredita-se estar livre de perigo.
>>
>
>

-- 
Esta mensagem foi verificada pelo sistema de antiv�rus e
 acredita-se estar livre de perigo.



[obm-l] Re: [obm-l] Re: [obm-l] Re: [obm-l] Polinômios

2017-07-25 Por tôpico Bruno Visnadi
Opa, deixei passar um erro bem básico! Estou corrigindo, um momentinho

Em 25 de julho de 2017 22:04, Pedro Júnior <pedromatematic...@gmail.com>
escreveu:

> Obrigado, didático e criativo.
> Valeu mesmo!
>
> Em 25 de jul de 2017 9:55 PM, "Bruno Visnadi" <brunovisnadida...@gmail.com>
> escreveu:
>
>> Seja P(x) = (x-2)(x-3)(x-4) = x³ - 9x² + 26x - 24 -> P(1) = -6
>>
>> Seja Q(x) = P(x) + 6 -> Q(1) = 0 -> Q(x) é múltiplo de (x-1)
>>
>> Perceba que Q(x) deixa resto 6 por (x-2), (x-3) e (x-4). Todo polinômio
>> no formato Q(x) + n*P(x), para todo n, deixa resto 6 por
>> (x-2), (x-3) e (x-4).
>>
>> Em 25 de julho de 2017 21:22, Pedro Júnior <pedromatematic...@gmail.com>
>> escreveu:
>>
>>> Prove que existem infinitos polinômios de grau 3 de coeficientes reais
>>> que são divisíveis por x - 1 e que deixam o mesmo resto por x - 2, x - 3 e
>>> x - 4.
>>>
>>> Quem tiver uma boa dica fica meus agradecimentos.
>>>
>>> --
>>> Esta mensagem foi verificada pelo sistema de antivírus e
>>> acredita-se estar livre de perigo.
>>
>>
>>
>> --
>> Esta mensagem foi verificada pelo sistema de antivírus e
>> acredita-se estar livre de perigo.
>
>
> --
> Esta mensagem foi verificada pelo sistema de antivírus e
> acredita-se estar livre de perigo.
>

-- 
Esta mensagem foi verificada pelo sistema de antiv�rus e
 acredita-se estar livre de perigo.



[obm-l] Re: [obm-l] Polinômios

2017-07-25 Por tôpico Bruno Visnadi
Seja P(x) = (x-2)(x-3)(x-4) = x³ - 9x² + 26x - 24 -> P(1) = -6

Seja Q(x) = P(x) + 6 -> Q(1) = 0 -> Q(x) é múltiplo de (x-1)

Perceba que Q(x) deixa resto 6 por (x-2), (x-3) e (x-4). Todo polinômio no
formato Q(x) + n*P(x), para todo n, deixa resto 6 por
(x-2), (x-3) e (x-4).

Em 25 de julho de 2017 21:22, Pedro Júnior 
escreveu:

> Prove que existem infinitos polinômios de grau 3 de coeficientes reais que
> são divisíveis por x - 1 e que deixam o mesmo resto por x - 2, x - 3 e x -
> 4.
>
> Quem tiver uma boa dica fica meus agradecimentos.
>
> --
> Esta mensagem foi verificada pelo sistema de antivírus e
> acredita-se estar livre de perigo.

-- 
Esta mensagem foi verificada pelo sistema de antiv�rus e
 acredita-se estar livre de perigo.



Re: [obm-l] Probabilidade

2017-07-25 Por tôpico Bruno Visnadi
Concordo, Marcelo. De fato, a última metade da minha solução está
incorreta. A probabilidade de um subconjunto específico de K elementos
sobrar é de fato [(N-K)!*(N-A)!/(N!*(N-K-A)!)]^P, mas é possível que outros
números não pertencentes a este subconjunto tenham sobrado!

Então, a probabilidade de somente estes K números sobrarem é
[(N-K)!*(N-A)!/(N!*(N-K-A)!)]^P
- soma de X = K+1 até N-A:
{(N-K)!*[(N-X)!*(N-A)!/(N!*(N-X-A)!)]^P}/((X-K)!*(N-X-K)!).
Isto é, descontei todos os casos em que sobram mais números. Bom, vamos
chamar isto tudo de B.

Então, enfim, as chances de quaisquer K números sobrarem é B*N!/(K!*(N-K)!)

Outra condição que esqueci de mencionar, e que é necessária para que a
fórmula funcione, é K ≥ N - P*A, caso contrário provavelmente o resultado
sairá negativo (enquanto deveria ser 0).


Em 25 de julho de 2017 03:20, Marcelo Salhab Brogliato <msbro...@gmail.com>
escreveu:

> Oi Pedro e Bruno,
>
> K é só a quantidade de números que sobram (podendo ser quaisquer números
> do intervalo).
>
> Vejam o seguinte caso particular: N=10, A=2, P=4, K=3.
> Nesse caso, serão escolhidos 4 pares (a, b), a != b, ou seja, um total de
> 8 números no intervalo [1, 10].
>
> Pela equação de vocês:
> [1] comb(N-K, A) = comb(7, 2) = 21
> [2] comb(N, A) = comb(10, 2) = 45
> [3] comb(N, K) = comb(10, 3) = 120
>
> Assim, a probabilidade de pelo menos K=3 números não serem escolhidos
> seria: 120 * (21/45)^P = 120 * (21/45)^4 = 120 * 0.04742 = 5.6912 = 569.12%.
>
> O que eu acho que está errado na solução de vocês é que não podemos
> multiplicar por comb(N, K), pois ele irá "contar várias vezes o mesmo
> caso". Por exemplo: Quando os números 1, 2 e 3 foram retirados da seleção,
> a probabilidade parece ser (21/45)^4. Mas, nessas combinações, aconteceu o
> caso em que o número 4 não foi escolhido também. Esse caso em que não
> aparecem os números 1, 2, 3 e 4, também se repete quando os números
> retirados são 1, 2 e 4, pois, em algum momento, o 3 não será escolhido.
> Esse é só um caso de repetição dentre muitos. Concordam?
>
> Abraços,
> Salhab
>
> 2017-07-25 1:03 GMT-03:00 Bruno Visnadi <brunovisnadida...@gmail.com>:
>
>> Bom, se tirar a parte que eu multiplico por N!/(K!*(N-K)!), acho que
>> fica igual ao seu :)
>>
>> Realmente pelo enunciado não dá para saber se K é só a quantidade de
>> números que sobram, ou se são K números específicos.
>>
>> Em 24 de julho de 2017 23:37, Pedro Angelo <pedro.fon...@gmail.com>
>> escreveu:
>>
>>> Eu e o Bruno claramente entendemos o problema de forma diferente hehehe.
>>> Eu tava achando que os K números não deviam ser escolhidos eram K números
>>> pré-determinados (fixos). Eu entendi que "esses K números aqui não devem
>>> ser escolhidos", enquanto o Bruno entendeu que "Retirando dos N números os
>>> números que foram escolhidos, devem sobrar K números."
>>> ​
>>>
>>> --
>>> Esta mensagem foi verificada pelo sistema de antivírus e
>>> acredita-se estar livre de perigo.
>>>
>>
>>
>> --
>> Esta mensagem foi verificada pelo sistema de antivírus e
>> acredita-se estar livre de perigo.
>>
>
>
> --
> Esta mensagem foi verificada pelo sistema de antivírus e
> acredita-se estar livre de perigo.
>

-- 
Esta mensagem foi verificada pelo sistema de antiv�rus e
 acredita-se estar livre de perigo.



Re: [obm-l] Probabilidade

2017-07-24 Por tôpico Bruno Visnadi
Bom, se tirar a parte que eu multiplico por N!/(K!*(N-K)!), acho que fica
igual ao seu :)

Realmente pelo enunciado não dá para saber se K é só a quantidade de
números que sobram, ou se são K números específicos.

Em 24 de julho de 2017 23:37, Pedro Angelo 
escreveu:

> Eu e o Bruno claramente entendemos o problema de forma diferente hehehe.
> Eu tava achando que os K números não deviam ser escolhidos eram K números
> pré-determinados (fixos). Eu entendi que "esses K números aqui não devem
> ser escolhidos", enquanto o Bruno entendeu que "Retirando dos N números os
> números que foram escolhidos, devem sobrar K números."
> ​
>
> --
> Esta mensagem foi verificada pelo sistema de antivírus e
> acredita-se estar livre de perigo.
>

-- 
Esta mensagem foi verificada pelo sistema de antiv�rus e
 acredita-se estar livre de perigo.



Re: [obm-l] Probabilidade

2017-07-24 Por tôpico Bruno Visnadi
Condição: K + A < N, sendo todos inteiros positivos.

Podemos pensar assim:

Qual é a probabilidade de os números 1, 2, 3... K não serem escolhidos por
ninguém?
Sobram N - K números para cada pessoa escolher. Então cada uma tem
(N-K)!/(A!*(N-K-A)!) maneiras de escolher estes números, de um total de
N!/(A!(N-A)!. A probabilidade de uma pessoa escolher somente números
maiores que K é, portanto, (N-K)!*A!*(N-A)!/A!*(N-K-A)!*N! =
(N-K)!*(N-A)!/N!*(N-K-A)!. A chance de todas elas fazerem isso é este
número elevado a P, ou seja, [(N-K)!*(N-A)!/N!*(N-K-A)!]^P
Esta é a chance de os primeiros K números sobrarem. Como existem
N!/(K!*(N-K)!) maneiras de escolher K números, devemos multiplicá-la por
este número, e obtemos:

[(N-K)!*(N-A)!/N!*(N-K-A)!]^P * N!/(K!*(N-K)!)

Esta é, porém, a chance de PELO MENOS K números sobrarem. A chance de
exatamente K números sobrarem será:

[(N-K)!*(N-A)!/N!*(N-K-A)!]^P * N!/(K!*(N-K)!) - (somatória de T = K+1 a
N-A [(N-T)!*(N-A)!/N!*(N-T-A)!]^P * N!/(T!*(N-T)!)

Acho que é isso!

Em 24 de julho de 2017 18:27, Pedro Angelo 
escreveu:

> Oi Salhab!
>
> Pensei numas coisas elementares aqui, não sei o quão fechada é a fórmula
> que vc quer.
>
> A probabilidade de um dos K números não ser o primeiro dos A números
> escolhidos pela primeira das P pessoas é (N-1)/N. Dado que esse número de
> fato não foi o primeiro escolhido, a probabilidade de ele não ter sido o
> segundo escolhido é (N-2)/N. Continuando, a probabilidade de ele não ter
> sido nenhum dos A números escolhidos é (N-1)!/[N^A * (A-1)!]. A
> probabilidade de esse número não ter sido escolhido por nenhuma das P
> pessoas, já que cada escolhe os A números de forma independente, é
> simplesmente isso aí elevado a P:
>
> { (N-1)! / [N^A * (A-1)!] } ^ P
>
>
> Agora, dado que o primeiro dos K números não foi escolhido por nenhuma
> pessoa, a probabilidade de o segundo dos K números também não ter sido
> escolhido é dada pela mesma fórmula aí de cima, mas trocando N por N-1, já
> que sabemos que esse negundo número é diferente do primeiro (ou seja, o
> problema é o mesmo, mas eliminando um dos N números). Continuando, a
> resposta fica:
>
> { (N-1)! (N-2)! ... (N-K)! / [ (N!/(N-K)!)^A (A-1)!^K ] } ^ P
>
> Posso ter cometido algum engano (ou vários hehe), mas não sei se dá pra
> chegar a um resultado mais simples que esse.
>
> abraços
>
>
> 2017-07-24 17:52 GMT-03:00 Marcelo Salhab Brogliato :
>
>> Pessoal,
>>
>> Estou tentando resolver o seguinte problema:
>>
>> Dado que P pessoas selecionam aleatoriamente A>=2 inteiros diferentes no
>> intervalo [1, N], qual a probabilidade de K números do intervalo [1, N] não
>> serem selecionados por ninguém?
>>
>> Alguém pode me ajudar? :)
>>
>> Abraços,
>> Salhab
>>
>> --
>> Esta mensagem foi verificada pelo sistema de antivírus e
>> acredita-se estar livre de perigo.
>
>
>
> --
> Esta mensagem foi verificada pelo sistema de antivírus e
> acredita-se estar livre de perigo.
>

-- 
Esta mensagem foi verificada pelo sistema de antiv�rus e
 acredita-se estar livre de perigo.



Re: [obm-l] Problema estranho

2017-07-08 Por tôpico Bruno Visnadi
Tecnicamente não dá para chamar de conjunto, quando há números repetidos. O
correto seria Multiconjunto: https://pt.wikipedia.org/wiki/Multiconjunto

Em 8 de julho de 2017 19:27, Luiz Antonio Rodrigues 
escreveu:

> Olá, Otávio!
> Desculpe a intromissão. Eu não sei como resolver seu problema, mas quero
> aproveitá-lo para colocar uma questão que me atormenta desde a faculdade:
> pode existir um conjunto {1,1,1,2,3}? O número 1 não é único?
> Um abraço!
> Luiz
>
> On Jul 8, 2017 5:35 PM, "Otávio Araújo"  wrote:
>
> Galera, queria que alguém pudesse resolver essa questão pra mim ( passei
> muito tempo nela já kkk):
> " Seja n um natural positivo e A um conjunto de 2n+1 números reais, não
> necessariamente distintos, com a seguinte propriedade:
> - Todo subconjunto de A com 2n elementos pode ser particionado em dois
> conjuntos de n elementos tais que a soma dos elementos de cada um desses
> dois conjuntos de n elementos são iguais.
>Prove que todos os elementos de A são iguais."
>
>
>
>
>
>
>
> --
> Esta mensagem foi verificada pelo sistema de antivírus e
>  acredita-se estar livre de perigo.
>
>
> =
> Instruções para entrar na lista, sair da lista e usar a lista em
> http://www.mat.puc-rio.br/~obmlistas/obm-l.html
> =
>
>
>
> --
> Esta mensagem foi verificada pelo sistema de antivírus e
> acredita-se estar livre de perigo.
>

-- 
Esta mensagem foi verificada pelo sistema de antiv�rus e
 acredita-se estar livre de perigo.



Re: [obm-l] boatos sobre elon lages lima

2017-05-23 Por tôpico Bruno Visnadi
https://pt.wikipedia.org/wiki/Elon_Lages_Lima
A data que consta na página da Wikipedia deve ser correta.

Em 23 de maio de 2017 21:00, Israel Meireles Chrisostomo <
israelmchrisost...@gmail.com> escreveu:

> É verdade que o Elon morreu?Fiquei chocado com essa notícia, o pessoal
> aqui poderia confirmar a veracidade dessa notícia?
>
> --
> Esta mensagem foi verificada pelo sistema de antivírus e
> acredita-se estar livre de perigo.

-- 
Esta mensagem foi verificada pelo sistema de antiv�rus e
 acredita-se estar livre de perigo.



Re: [obm-l] Problema

2017-04-15 Por tôpico Bruno Visnadi
Bom, o que importa não é quantas vezes elas comem por dia, e sim o quanto
elas comem durante cada dia. Digamos que todas as 16 vacas juntas comam N
quilos de ração por dia, e temos 62N quilos ao total. Após 14 dias, sobram
48N quilos. Então ele vende 4 vacas, e a taxa de consumo passa a ser 3N/4
quilos por dia. Em 15 dias, isso dá 45N/4, e sobram 147N/4. Então chegam as
9 vacas, e a taxa de consumo passa a ser 21N/16 quilos por dia. Então, a
ração vai durar por (147N/4)/(21N/16) = 28 dias.

Em 15 de abril de 2017 10:21, Luiz Antonio Rodrigues 
escreveu:

> Olá, pessoal! Bom dia! Eu resolvi o problema abaixo supondo que as vacas
> comem uma vez por dia. Escrevi para perguntar se alguém consegue resolver
> de outra forma. A resposta é 28 dias. Muito obrigado, um abraço e uma ótima
> Páscoa para todos.
> Um fazendeiro possui ração suficiente para alimentar suas 16 vacas durante
> 62 dias. Após 14 dias, ele vende 4 vacas. Passados mais 15 dias, ele compra
> 9 vacas. Quantos dias vai durar o restante da ração?
>
> --
> Esta mensagem foi verificada pelo sistema de antivírus e
> acredita-se estar livre de perigo.

-- 
Esta mensagem foi verificada pelo sistema de antiv�rus e
 acredita-se estar livre de perigo.



[obm-l] Re: [obm-l] Re: [obm-l] Qual a maior potência?

2017-01-16 Por tôpico Bruno Visnadi
5^3 < 128 = 4^3.5

Então, 4^53/5^44 = 4^53/(5^2)*(5^42) > 4^53/25*4^49 = 4^4/25 > 1
Portanto, 4^53 > 5^44

Em 16 de janeiro de 2017 15:01, Esdras Muniz 
escreveu:

> 4^53 = 2^106 > 2^105 = (2^7)^15 = (128)^15 > 125^15 = 5^45 > 5^44.
>
> Em 16 de janeiro de 2017 13:14, Douglas Oliveira de Lima <
> profdouglaso.del...@gmail.com> escreveu:
>
>> Olá amigos , gostaria de uma ajuda pra um raciocínio diferente, por log
>> eu já fiz.
>>
>> Qual a maior potência? 4^53 ou 5^44.
>>
>> --
>> Esta mensagem foi verificada pelo sistema de antivírus e
>> acredita-se estar livre de perigo.
>
>
>
>
> --
> Esdras Muniz Mota
> Mestrando em Matemática
> Universidade Federal do Ceará
>
>
>
> --
> Esta mensagem foi verificada pelo sistema de antivírus e
> acredita-se estar livre de perigo.
>

-- 
Esta mensagem foi verificada pelo sistema de antiv�rus e
 acredita-se estar livre de perigo.



Re: [obm-l] Combinatoria

2016-12-26 Por tôpico Bruno Visnadi
Considerando os dígitos (0, 1, 2, 3, 4, 5, 6, 7), temos duas possibilidades:
Ou dois dos cadeados tem algum dos dígitos (0, 1, 2, 3), ou dois deles tem
algum dos dígitos (4, 5, 6, 7).
Vamos supor que ao menos 2 deles tenham dígitos do grupo (0, 1, 2, 3).
Então queremos cobrir todas as possibilidades que envolvem estes dígitos.
Cada tentativa (A, B, C) elimina 3 possibilidades (A, B, X), (A, X, C) e
(X, B, C). Existem, ao todo, 48 possibilidades (16 pares possíveis, e 3
posições). Portanto precisamos de 48/3 = 16 tentativas, desde que não se
repita nenhum par. E isto é possível:
(0, 0, 0), (0, 1, 1), (0, 2, 2), (0, 3, 3), (1, 0, 1), (1, 1, 2), (1, 2,
3), (1, 3, 0), (2, 0, 2), (2, 1, 3), (2, 2, 0), (2, 3, 1), (3, 0, 3), (3,
1, 0), (3, 2, 1), (3, 3, 2).
Se nossa suposição estava errada, e na verdade 2 dos cadeados tem dígitos
do grupo (4, 5, 6, 7), basta fazer as mesmas tentativas, mas trocando os
dígitos (0, 1, 2, 3) por (4, 5, 6, 7). Assim, abriremos o armário em no
máximo 32 tentativas.

Em 26 de dezembro de 2016 08:26, Pedro José  escreveu:

> Bom dia!
>
> Fui inocente, fiz uma restrição que não precisava. Não há necessidade de
> acaso.
> Pode haver estratégia.
>
> Saudações,
> PJMS
>
> Em 25 de dezembro de 2016 12:31, Matheus Herculano <
> matheusherculan...@gmail.com> escreveu:
>
>> 87
>>
>> Em 23 de dez de 2016 13:07, "Gabriel Tostes" 
>> escreveu:
>>
>>> Um armario de segurança tem 3 cadeados. Cada cadeado tem 8 combinacoes
>>> diferentes. O armario abre se quaisquer 2 dos 3 cadeados estao na posicao
>>> correta, qual e o numero minimo de tentativas pra abrir o armario?
>>>
>>>
>>> --
>>> Esta mensagem foi verificada pelo sistema de antivírus e
>>>  acredita-se estar livre de perigo.
>>>
>>>
>>> 
>>> =
>>> Instruções para entrar na lista, sair da lista e usar a lista em
>>> http://www.mat.puc-rio.br/~obmlistas/obm-l.html
>>> 
>>> =
>>>
>>
>> --
>> Esta mensagem foi verificada pelo sistema de antivírus e
>> acredita-se estar livre de perigo.
>>
>
>
> --
> Esta mensagem foi verificada pelo sistema de antivírus e
> acredita-se estar livre de perigo.
>

-- 
Esta mensagem foi verificada pelo sistema de antiv�rus e
 acredita-se estar livre de perigo.



Re: [obm-l] Combinatoria

2016-12-23 Por tôpico Bruno Visnadi
Ops, que viagem, é mesmo.
Mas certamente é um valor menor que 64. Se fizéssemos com 4x4x4 a
estratégia de deixar um fixo e alterar os outros, dariam 16 possibilidades,
mas são apenas 8.

Em 23 de dezembro de 2016 16:41, Gabriel Tostes 
escreveu:

> 24 nao eh possivel...
>
> > On Dec 23, 2016, at 16:22, Ralph Teixeira  wrote:
> >
> > Hm, acho que dah para fazer com menos tentativas.
> >
> > Sejam a, b e c as combinacoes corretas de cada cadeado, onde a,b,c
> > estao em {0,1,2,3,4,5,6,7}.
> >
> > Tentanto, por exemplo, todas as combinacoes possiveis para a e b
> > (mantenha c=0), fazemos 64 tentativas, e com certeza vamos acabar
> > acertando a combinacao dos dois primeiros cadeados -- o que eh
> > suficiente para abrir o armario!
> >
> > Mas eu nao estou dizendo que a resposta eh 64 -- acho que dah para ser
> > mais esperto e abrir o armario garantidamente com menos tentativas...
> >
> > (24, talvez?)
> >
> > Abraco, Ralph.
> >
> > P.S.: Pode me chamar de maluco, mas eu estou enxergando um cubo
> > dividido em 8x8x8 cubinhos de LED, e a combinacao correta eh um
> > cubinho especial desconhecido. Os 512 cubinhos comecam apagados; cada
> > vez que voce faz uma tentativa, voce estah escolhendo um cubinho, e
> > acendendo nao soh ele, mas todos os cubinhos na mesma linha, coluna
> > ou... huh, outra linha. Em outras palavras, se voce escolhe o cubinho
> > (A,B,C) (eu imagino voce botando o dedo nele para acende-lo, como se
> > fosse uma jogada de um joguinho), voce acende todos os 22 cubinhos da
> > forma (A,B,x), (A,x,C) ou (x,B,C) onde 0<=x<=7. Digo isso porque, se a
> > combinacao correta dos 3 cadeados fosse um dos que acendeu, voce teria
> > acertado pelo menos 2 cadeados, e assim abria o armario; e vice-versa,
> > voce soh acerta se o cubinho desconhecido estiver entre esses 22.
> >
> > Entao o problema eh o seguinte: qual a maneira mais economica (menos
> > jogadas) de acender todos os 512 cubinhos no meu joguinho de LEDs? Eh,
> > vai ter que acender **todos**, porque se voce esquecer unzinho, podia
> > dar azar e ser aquela a combinacao correta, e entao voce nao garante
> > abrir o armario!
> >
> > Obviamente, como cada jogada acende 22, e sao 512 cubinhos, vamos
> > precisar de no minimo 512/22, huh, arredonda para cima, 24 jogadas.
> > Mas dah para fazer com 24? Para tanto, voce teria que ter muito poucas
> > intersecoes entre jogadas distintas -- eh possivel?
> >
> > 2016-12-23 14:53 GMT-02:00 Pedro José :
> >> Bom dia!
> >>
> >> Novamente o problema está mal formulado.
> >> Embora possa parecer claro, qual é o número mínimo  de tentativas que
> >> garanta abrir o armário.
> >>
> >> Dois casos disjuntos atendem.
> >>
> >> (i) Dois cadeados corretos e o outro errado.
> >>
> >> Há uma chance de cada cadeado estar correto e 7 chances do terceiro
> estar
> >> errado. Há 3 = C(3,2) jeitos de distribuir os dois cadeados corretos e
> o
> >> errado.
> >>
> >> Pelo princípio da multiplicação são: 3*7 = 21 eventos.
> >>
> >> (ii) os três cadeados estão corretos;
> >>
> >> Só há uma possibilidade.
> >>
> >> O total de possibilidades para estar correto são 22 eventos.
> >>
> >> O universo tem 8^3, logo há 8^3 -22 possibilidades que não abrem o
> armário.
> >>
> >> Portanto para garantir que abra teremos 8^3 -22 +1 = 8^3 -21 = 491
> >> tentativas.
> >>
> >> Mas do jeito que o problema está formulado é 1. Se a pessoa der sorte
> de
> >> acertar de primeira.
> >>
> >> Saudações,
> >> PJMS
> >>
> >>
> >>
> >>
> >>
> >> Em 23 de dezembro de 2016 11:53, Gabriel Tostes 
> >> escreveu:
> >>>
> >>> Um armario de segurança tem 3 cadeados. Cada cadeado tem 8
> combinacoes
> >>> diferentes. O armario abre se quaisquer 2 dos 3 cadeados estao na
> posicao
> >>> correta, qual e o numero minimo de tentativas pra abrir o armario?
> >>>
> >>>
> >>> --
> >>> Esta mensagem foi verificada pelo sistema de antivírus e
> >>> acredita-se estar livre de perigo.
> >>>
> >>>
> >>> 
> =
> >>> Instruções para entrar na lista, sair da lista e usar a lista em
> >>> http://www.mat.puc-rio.br/~obmlistas/obm-l.html
> >>> 
> =
> >>
> >>
> >>
> >> --
> >> Esta mensagem foi verificada pelo sistema de antivírus e
> >> acredita-se estar livre de perigo.
> >
> > --
> > Esta mensagem foi verificada pelo sistema de antivírus e
> > acredita-se estar livre de perigo.
> >
> >
> > 
> =
> > Instruções para entrar na lista, sair da lista e usar a lista em
> > http://www.mat.puc-rio.br/~obmlistas/obm-l.html
> > 
> =
>
> --
> Esta mensagem foi verificada pelo sistema de antivírus e
>  acredita-se estar livre de perigo.
>
>
> 

Re: [obm-l] Combinatoria

2016-12-23 Por tôpico Bruno Visnadi
Gostei da analogia dos cubinhos, mas não acredito que a resposta seja 24.
Lembre-se que os últimos cubinhos escolhidos vão acender menos cubinhos,
pois alguns já estarão acesos.
Estava tentando com menos possibilidades por cadeado. Se fossem 4, por
exemplo, seria possível abrir o cadeado certamente escolhendo as seguintes
combinações:
(1, 1, 1)
(1, 3, 3)
(2, 2, 2)
(2, 4, 4)
(3, 1, 3)
(3, 3, 1)
(4, 4, 2)
(4, 2, 4)
São 8 combinações ao todo. Como um cubo 8x8x8 pode ser montado com 4 cubos
4x4x4, poderíamos certamente cobrir todas as possibilidades com 4*8 = 32
tentativas. Portanto, a resposta é menor ou igual a 32.


Em 23 de dezembro de 2016 16:22, Ralph Teixeira 
escreveu:

> Hm, acho que dah para fazer com menos tentativas.
>
> Sejam a, b e c as combinacoes corretas de cada cadeado, onde a,b,c
> estao em {0,1,2,3,4,5,6,7}.
>
> Tentanto, por exemplo, todas as combinacoes possiveis para a e b
> (mantenha c=0), fazemos 64 tentativas, e com certeza vamos acabar
> acertando a combinacao dos dois primeiros cadeados -- o que eh
> suficiente para abrir o armario!
>
> Mas eu nao estou dizendo que a resposta eh 64 -- acho que dah para ser
> mais esperto e abrir o armario garantidamente com menos tentativas...
>
> (24, talvez?)
>
> Abraco, Ralph.
>
> P.S.: Pode me chamar de maluco, mas eu estou enxergando um cubo
> dividido em 8x8x8 cubinhos de LED, e a combinacao correta eh um
> cubinho especial desconhecido. Os 512 cubinhos comecam apagados; cada
> vez que voce faz uma tentativa, voce estah escolhendo um cubinho, e
> acendendo nao soh ele, mas todos os cubinhos na mesma linha, coluna
> ou... huh, outra linha. Em outras palavras, se voce escolhe o cubinho
> (A,B,C) (eu imagino voce botando o dedo nele para acende-lo, como se
> fosse uma jogada de um joguinho), voce acende todos os 22 cubinhos da
> forma (A,B,x), (A,x,C) ou (x,B,C) onde 0<=x<=7. Digo isso porque, se a
> combinacao correta dos 3 cadeados fosse um dos que acendeu, voce teria
> acertado pelo menos 2 cadeados, e assim abria o armario; e vice-versa,
> voce soh acerta se o cubinho desconhecido estiver entre esses 22.
>
> Entao o problema eh o seguinte: qual a maneira mais economica (menos
> jogadas) de acender todos os 512 cubinhos no meu joguinho de LEDs? Eh,
> vai ter que acender **todos**, porque se voce esquecer unzinho, podia
> dar azar e ser aquela a combinacao correta, e entao voce nao garante
> abrir o armario!
>
> Obviamente, como cada jogada acende 22, e sao 512 cubinhos, vamos
> precisar de no minimo 512/22, huh, arredonda para cima, 24 jogadas.
> Mas dah para fazer com 24? Para tanto, voce teria que ter muito poucas
> intersecoes entre jogadas distintas -- eh possivel?
>
> 2016-12-23 14:53 GMT-02:00 Pedro José :
> > Bom dia!
> >
> > Novamente o problema está mal formulado.
> > Embora possa parecer claro, qual é o número mínimo  de tentativas que
> > garanta abrir o armário.
> >
> > Dois casos disjuntos atendem.
> >
> > (i) Dois cadeados corretos e o outro errado.
> >
> > Há uma chance de cada cadeado estar correto e 7 chances do terceiro estar
> > errado. Há 3 = C(3,2) jeitos de distribuir os dois cadeados corretos e o
> > errado.
> >
> > Pelo princípio da multiplicação são: 3*7 = 21 eventos.
> >
> > (ii) os três cadeados estão corretos;
> >
> > Só há uma possibilidade.
> >
> > O total de possibilidades para estar correto são 22 eventos.
> >
> > O universo tem 8^3, logo há 8^3 -22 possibilidades que não abrem o
> armário.
> >
> > Portanto para garantir que abra teremos 8^3 -22 +1 = 8^3 -21 = 491
> > tentativas.
> >
> > Mas do jeito que o problema está formulado é 1. Se a pessoa der sorte de
> > acertar de primeira.
> >
> > Saudações,
> > PJMS
> >
> >
> >
> >
> >
> > Em 23 de dezembro de 2016 11:53, Gabriel Tostes 
> > escreveu:
> >>
> >> Um armario de segurança tem 3 cadeados. Cada cadeado tem 8 combinacoes
> >> diferentes. O armario abre se quaisquer 2 dos 3 cadeados estao na
> posicao
> >> correta, qual e o numero minimo de tentativas pra abrir o armario?
> >>
> >>
> >> --
> >> Esta mensagem foi verificada pelo sistema de antivírus e
> >>  acredita-se estar livre de perigo.
> >>
> >>
> >> 
> =
> >> Instruções para entrar na lista, sair da lista e usar a lista em
> >> http://www.mat.puc-rio.br/~obmlistas/obm-l.html
> >> 
> =
> >
> >
> >
> > --
> > Esta mensagem foi verificada pelo sistema de antivírus e
> > acredita-se estar livre de perigo.
>
> --
> Esta mensagem foi verificada pelo sistema de antivírus e
>  acredita-se estar livre de perigo.
>
>
> =
> Instru�ões para entrar na lista, sair da lista e usar a lista em
> http://www.mat.puc-rio.br/~obmlistas/obm-l.html
> =
>

-- 
Esta mensagem foi 

Re: [obm-l] Re: Problema de geometria.

2016-11-02 Por tôpico Bruno Visnadi
Fiz um esquema no paint da figura, para ficar mais claro. Em vermelho são
as flechas, que ligam o ponto médio do lado ao ponto médio do arco
determinado pelo lado.

Em 2 de novembro de 2016 20:22, Douglas Oliveira de Lima <
profdouglaso.del...@gmail.com> escreveu:

> Me desculpe pela ignorância, deixa eu detalhar para não gerar filosofias
> vãs.
> Dado um triângulo ABC inscrito em uma  circunferência, e os pontos médios
> de seus lados, M ponto médio do lado BC,
> N ponto médio do lado AC e P ponto médio do lado AB, considere agora os
> pontos médios D, E, F dos menores arcos (AB), (AC), (BC) respectivamente,
> se os segmentos DP=1 cm, EN=2 cm e MF=3 cm, calcule a área do triângulo.
>
>
> Em 2 de novembro de 2016 18:57, Douglas Oliveira de Lima <
> profdouglaso.del...@gmail.com> escreveu:
>
>> Olá amigos , preciso de uma ajuda na seguinte questão, na verdade a
>> resolução porque já tentei muita coisa, já aprendi muita coisa com ela, mas
>> mesmo assim não a resolvi.
>>
>> As três flechas dos três lados (cordas) de um triângulo ABC inscrito em
>> uma circunferência de raio R
>> valem 1, 2 e 3 calcular a área do triângulo.
>>
>>
>>
>> Qualquer ajuda será bem vinda. Obrigado.
>>
>> Att . Douglas Oliveira
>>
>
>
> --
> Esta mensagem foi verificada pelo sistema de antivírus e
> acredita-se estar livre de perigo.
>

-- 
Esta mensagem foi verificada pelo sistema de antiv�rus e
 acredita-se estar livre de perigo.



Re: [obm-l] Problema de geometria.

2016-11-02 Por tôpico Bruno Visnadi
De acordo com o site
http://objetoseducacionais2.mec.gov.br/bitstream/handle/mec/10396/geo0500.htm
Flecha é um segmento de reta que une o ponto médio de uma corda ao ponto
médio do arco correspondente.


Em 2 de novembro de 2016 20:06, Tarsis Esau  escreveu:

> Se essas "flechas" forem lados o triângulo não existe.
>
> Em 02/11/2016 6:16 PM, "Esdras Muniz" 
> escreveu:
>
>> O que são essas "flechas"?
>>
>> Em 2 de novembro de 2016 17:57, Douglas Oliveira de Lima <
>> profdouglaso.del...@gmail.com> escreveu:
>>
>>> Olá amigos , preciso de uma ajuda na seguinte questão, na verdade a
>>> resolução porque já tentei muita coisa, já aprendi muita coisa com ela, mas
>>> mesmo assim não a resolvi.
>>>
>>> As três flechas dos três lados (cordas) de um triângulo ABC inscrito em
>>> uma circunferência de raio R
>>> valem 1, 2 e 3 calcular a área do triângulo.
>>>
>>>
>>>
>>> Qualquer ajuda será bem vinda. Obrigado.
>>>
>>> Att . Douglas Oliveira
>>>
>>> --
>>> Esta mensagem foi verificada pelo sistema de antivírus e
>>> acredita-se estar livre de perigo.
>>
>>
>>
>>
>> --
>> Esdras Muniz Mota
>> Mestrando em Matemática
>> Universidade Federal do Ceará
>>
>>
>>
>> --
>> Esta mensagem foi verificada pelo sistema de antivírus e
>> acredita-se estar livre de perigo.
>
>
> --
> Esta mensagem foi verificada pelo sistema de antivírus e
> acredita-se estar livre de perigo.
>

-- 
Esta mensagem foi verificada pelo sistema de antiv�rus e
 acredita-se estar livre de perigo.



[obm-l] Re: [obm-l] Re: [obm-l] nome de um quadrilátero

2016-08-10 Por tôpico Bruno Visnadi
De acordo com o próprio Wikipédia, o nome é 'Deltoide' ou 'Pipa'.

Em 10 de agosto de 2016 18:43, Luís Lopes  escreveu:

> Sauda,c~oes, oi Esdras,
>
> Obrigado. Difícil imaginar isso pois rhombus
>
> em inglês parece ser losango.
>
>
> https://pt.wikipedia.org/wiki/Losango
>
>
> Não me lembro de ter visto esse nome rombo.
>
>
> Os livros didáticos usam esse nome para kite ?
>
>
> Luís
>
> --
> *De:* owner-ob...@mat.puc-rio.br  em nome de
> Esdras Muniz 
> *Enviado:* quarta-feira, 10 de agosto de 2016 21:13:26
> *Para:* obm-l@mat.puc-rio.br
> *Assunto:* [obm-l] Re: [obm-l] nome de um quadrilátero
>
> Rombo.
>
> Em 10 de agosto de 2016 17:43, Luís Lopes 
> escreveu:
>
>> Sauda,c~oes,
>>
>>
>> Qual o nome em português para o
>>
>> quadrilátero chamado de kite em inglês ?
>>
>>
>>
>> https://en.wikipedia.org/wiki/Kite_(geometry)
>>
>>
>> Abs,
>>
>> Luís
>>
>>
>>
>> --
>> Esta mensagem foi verificada pelo sistema de antivírus e
>> acredita-se estar livre de perigo.
>>
>
>
>
> --
> Esdras Muniz Mota
> Mestrando em Matemática
> Universidade Federal do Ceará
>
>
>
> --
> Esta mensagem foi verificada pelo sistema de antiv�rus e
> acredita-se estar livre de perigo.
>
> --
> Esta mensagem foi verificada pelo sistema de antivírus e
> acredita-se estar livre de perigo.
>

-- 
Esta mensagem foi verificada pelo sistema de antiv�rus e
 acredita-se estar livre de perigo.



[obm-l] Re: [obm-l] Duas questões de matemática.

2016-08-08 Por tôpico Bruno Visnadi
Olá

Não sei responder sobre os ângulos suplementares.
Sobre o problema, não acho que ele esteja mal elaborado.
O total de dinheiro disputado é 750. Como ambos pagaram e receberam o
mesmo, cada um pagou e recebeu 375.
Como 15+20+25+30+35+40+45+50+55+60 = 65+70+75+80+85 = 375, é possível que
Ricardo tenha ganhado 10 partidas e perdido 5. É impossível que alguém
tenha ganhado 11, caso contrário a soma mínima seria 440, logo, a maior
diferença possível é 5, alternativa E.

Em 8 de agosto de 2016 16:45, Douglas Oliveira de Lima <
profdouglaso.del...@gmail.com> escreveu:

> Olá amigos, gostaria de uma ajuda em uma filosofia e uma questão.
>
> 1)Na definição de ângulos suplementares, seria para dois ângulos ou pode
> ser para mais de dois?
>
> 2)(Essa questão gostaria de saber se está mal elaborada) Carlos e Ricardo
> disputaram 15 partidas de boliche e ao fim de cada partida o perdedor
> pagava um prêmio em dinheiro para o vencedor. O prêmio para a primeira
> partida foi R$ 15,00 e o prê- mio de cada partida seguinte foi R$ 5,00 a
> mais do que o valor da partida anterior. Ao final da disputa, ambos
> receberam o mesmo valor em dinheiro e nenhuma partida terminou empatada.
> Nes- sas condições, a maior diferença possível entre as vitórias e as
> derrotas de Ricardo é
> (A) 4. (B) 3. (C) 7. (D) 6. (E) 5.
>
> Att: Douglas Oliveira.
>
> --
> Esta mensagem foi verificada pelo sistema de antivírus e
> acredita-se estar livre de perigo.

-- 
Esta mensagem foi verificada pelo sistema de antiv�rus e
 acredita-se estar livre de perigo.